GI (term 1 nursing)

Réussis tes devoirs et examens dès maintenant avec Quizwiz!

The nurse is observing a co-worker who is caring for a client with a nasogastric tube following esophageal surgery. Which actions by the co-worker require the nurse to intervene? (Select all that apply.) A. Checking tube placement every 12 hours B. Keeping the bed flat C. Placing the client upright when taking sips of water D. Providing mouth care every 8 hours E. Securing the tube

A. Checking tube placement every 12 hours B. Keeping the bed flat D. Providing mouth care every 8 hours The nasogastric tube should be checked every 4 to 8 hours. The head of the bed should be elevated at least 30 degrees. Oral hygiene should be provided every 2 to 4 hours. The client should be placed upright when taking sips or small amounts of water to prevent choking and to allow observation of the client for dysphagia. The tube should be secured to prevent dislodgment.

31. A nurse is providing discharge teaching for a patient who is going home with a guaiac test. Which statement by the patient indicates the need for further education? a. "If I get a positive result, I have gastrointestinal bleeding." b. "I should not eat red meat before my examination." c. "I should schedule to perform the examination when I am not menstruating." d. "I will need to perform this test three times if I have a positive result."

ANS: A A positive result does not mean GI bleeding; it could be a false positive from consuming red meat, some raw vegetables, or NSAIDs. Proper patient education is important for viable results. The patient needs to avoid certain foods to rule out a false positive. If the test is positive, the patient will need to repeat the test at least three times. Menses and hemorrhoids can also lead to false positives.

21. After a patient returns from a barium swallow, the nurse's priority is to a. Encourage the patient to increase fluids to flush out the barium. b. Monitor stools closely for bright red blood or mucus, which indicates trauma from the procedure. c. Inform the patient that his bowel movements are radioactive, and that he should be sure to flush the toilet three times. d. Thicken all patient drinks to prevent aspiration.

ANS: A Encourage the patient to increase fluid intake to flush and remove excess barium from the body. Barium swallow is a noninvasive procedure for which no trauma would produce blood or mucus or increase aspiration risk. Barium is not a radioactive substance, so multiple flushes are not needed.

A nurse assesses a client who is prescribed alosetron (Lotronex). Which assessment question should the nurse ask this client? a. Have you been experiencing any constipation? b. Are you eating a diet high in fiber and fluids? c. Do you have a history of high blood pressure? d. What vitamins and supplements are you taking?

ANS: A Ischemic colitis is a life-threatening complication of alosetron. The nurse should assess the client for constipation. The other questions do not identify complications related to alosetron.

3. Which of the following is not a function of the large intestine? a. Absorbing nutrients b. Absorbing water c. Secreting bicarbonate d. Eliminating waste

ANS: A Nutrient absorption is done in the small intestine. The other options are all functions of the large intestine.

2. The nurse would expect the least formed stool to be present in which portion of the digestive tract? a. Ascending b. Descending c. Transverse d. Sigmoid

ANS: A The path of digestion goes from the ascending, across the transverse, to the descending and finally passing into the sigmoid; therefore, the least formed stool would be in the ascending.

A nurse assesses a client with irritable bowel syndrome (IBS). Which questions should the nurse include in this clients assessment? (Select all that apply.) a. Which food types cause an exacerbation of symptoms? b. Where is your pain and what does it feel like? c. Have you lost a significant amount of weight lately? d. Are your stools soft, watery, and black in color? e. Do you experience nausea associated with defecation?

ANS: A, B, E The nurse should ask the client about factors that may cause exacerbations of symptoms, including food, stress, and anxiety. The nurse should also assess the location, intensity, and quality of the clients pain, and nausea associated with defecation or meals. Clients who have IBS do not usually lose weight and stools are not black in color.

7. A patient with a hip fracture is having difficulty defecating into a bed pan while lying in bed. Which action by the nurse would assist the patient in having a successful bowel movement? a. Administering laxatives to the patient b. Raising the head of the bed c. Preparing to administer a barium enema d. Withholding narcotic pain medication

ANS: B Lying in bed is an unnatural position; raising the head of the bed assists the patient into a more normal position that allows proper contraction of muscles for elimination. Laxatives would not give the patient control over bowel movements. A barium enema is a diagnostic test, not an intervention to promote defecation. Pain relief measures should be given; however, preventative action should be taken to prevent constipation.

1. The nurse knows that most nutrients are absorbed in which portion of the digestive tract? a. Stomach b. Duodenum c. Ileum d. Cecum

ANS: B Most nutrients are absorbed in the duodenum with the exception of certain vitamins, iron, and salt (which are absorbed in the ileum). Food is broken down in the stomach. The cecum is the beginning of the large intestine.

An emergency room nurse assesses a client after a motor vehicle crash and notes ecchymotic areas across the clients lower abdomen. Which action should the nurse take first? a. Measure the clients abdominal girth. b. Assess for abdominal guarding or rigidity. c. Check the clients hemoglobin and hematocrit. d. Obtain the clients complete health history.

ANS: B On noticing the ecchymotic areas, the nurse should check to see if abdominal guarding or rigidity is present, because this could indicate major organ injury. The nurse should then notify the provider. Measuring abdominal girth or obtaining a complete health history is not appropriate at this time. Laboratory test results can be checked after assessment for abdominal guarding or rigidity.

10. Which physiological change can cause a paralytic ileus? a. Chronic cathartic abuse b. Surgery for Crohn's disease and anesthesia c. Suppression of hydrochloric acid from medication d. Fecal impaction

ANS: B Surgical manipulation of the bowel can cause a paralytic ileus. The other options are incorrect.

A nurse cares for a client newly diagnosed with colon cancer who has become withdrawn from family members. Which action should the nurse take? a. Contact the provider and recommend a psychiatric consult for the client. b. Encourage the client to verbalize feelings about the diagnosis. c. Provide education about new treatment options with successful outcomes. d. Ask family and friends to visit the client and provide emotional support.

ANS: B The nurse recognizes that the client may be expressing feelings of grief. The nurse should encourage the client to verbalize feelings and identify fears to move the client through the phases of the grief process. A psychiatric consult is not appropriate for the client. The nurse should not brush aside the clients feelings with discussions related to cancer prognosis and treatment. The nurse should not assume that the client desires family or friends to visit or provide emotional support.

35. A nurse is caring for a patient who has had diarrhea for the past week. Which additional assessment finding would the nurse expect? a. Increased energy levels b. Distended abdomen c. Decreased serum bicarbonate d. Increased blood pressure

ANS: C ANS: C Chronic diarrhea can result in metabolic acidosis, which is diagnostic of low serum bicarbonate. Patients with chronic diarrhea are dehydrated with decreased blood pressure. Diarrhea also causes loss of electrolytes, nutrients, and fluid, which decreases energy levels. A distended abdomen would indicate constipation.

18. The nurse should question which order? a. A normal saline enema to be repeated every 4 hours until stool is produced b. A hypertonic solution enema with a patient with fluid volume excess c. A Kayexalate enema for a patient with hypokalemia d. An oil retention enema for a patient using mineral oil laxatives

ANS: C Kayexalate binds to and helps excrete potassium, so it would be contraindicated in patients who are hypokalemic (have low potassium). Normal saline enemas can be repeated without risk of fluid or electrolyte imbalance. Hypertonic solutions are intended for patients who cannot handle large fluid volume and are contraindicated for dehydrated patients. Because mineral oil laxatives and an oil retention enema have the same intended effect of lubricating the colon and rectum, an oil retention enema is not needed.

A nurse cares for a client with colon cancer who has a new colostomy. The client states, I think it would be helpful to talk with someone who has had a similar experience. How should the nurse respond? a. I have a good friend with a colostomy who would be willing to talk with you. b. The enterostomal therapist will be able to answer all of your questions. c. I will make a referral to the United Ostomy Associations of America. d. Youll find that most people with colostomies dont want to talk about them.

ANS: C Nurses need to become familiar with community-based resources to better assist clients. The local chapter of the United Ostomy Associations of America has resources for clients and their families, including Ostomates (specially trained visitors who also have ostomies). The nurse should not suggest that the client speak with a personal contact of the nurse. Although the enterostomal therapist is an expert in ostomy care, talking with him or her is not the same as talking with someone who actually has had a colostomy. The nurse should not brush aside the clients request by saying that most people with colostomies do not want to talk about them. Many people are willing to share their ostomy experience in the hope of helping others.

36. The nurse is caring for a patient who had a colostomy placed yesterday. The nurse should report which assessment finding immediately? a. Stoma is protruding from the abdomen. b. Stoma is moist. c. Stool is discharging from the stoma. d. Stoma is purple.

ANS: D A purple stoma may indicate strangulation or poor circulation to the stoma and may require surgical intervention. A stoma should be reddish-pink and moist in appearance. It can be flush with the skin, or it can protrude. Stool is an expected outcome of stoma placement.

12. The nurse provides knows that a bowel elimination schedule would be most beneficial in the plan of care for which patient? a. A 40-year-old patient with an ileostomy b. A 25-year-old patient with Crohn's disease c. A 30-year-old patient with C. difficile d. A 70-year-old patient with stool incontinence

ANS: D ANS: D A bowel elimination program is helpful for a patient with incontinence. It helps the person who still has neuromuscular control defecate normally. An ileostomy, Crohn's disease, and C. difficile all relate to uncontrollable bowel movements, for which no method can be used to set up a schedule of elimination.

An emergency room nurse cares for a client who has been shot in the abdomen and is hemorrhaging heavily. Which action should the nurse take first? a. Send a blood sample for a type and crossmatch. b. Insert a large intravenous line for fluid resuscitation. c. Obtain the heart rate and blood pressure. d. Assess and maintain a patent airway.

ANS: D All of the options are important nursing actions in the care of a trauma client. However, airway always comes first. The client must have a patent airway, or other interventions will not be helpful.

The nurse admits an immunocompromised client who has contracted herpes simplex stomatitis. The nurse anticipates that the health care provider will request which medication? A. Acyclovir (Zovirax) B. Diphenhydramine (Benadryl) C. Nystatin (Mycostatin) D. Tetracycline syrup (Sumycin syrup)

Acyclovir (Zovirax) Acyclovir (Zovirax) is an antiviral agent that is prescribed for immunocompromised clients who contract herpes simplex stomatitis. Diphenhydramine is an antihistamine that is not indicated for treating this condition. Nystatin is indicated for treatment of fungal infection. Tetracycline syrup is indicated for treatment of recurrent aphthous ulcers.

A client with oral carcinoma has a priority problem of risk for airway blockage related to obstruction by the tumor. At the beginning of the shift, which action will the nurse take first? A. Suction the client's oral secretions to clear the airway. B. Place the client on humidified oxygen per nasal cannula. C. Assist the client to an upright position to facilitate breathing. D. Assess the respiratory effort and quantities and types of oral secretions.

Assess the respiratory effort and quantities and types of oral secretions. Assessment is the first step of the nursing process; the nurse should assess the client's respiratory effort and quantities and types of oral secretions first. Suctioning the client, placing the client on humidified oxygen, and assisting the client to an upright position are not the first steps in the nursing process. These interventions may or may not be necessary if the nurse follows the nursing process.

The nurse is monitoring a client with gastric cancer for signs and symptoms of upper gastrointestinal bleeding. Which change in vital signs is most indicative of bleeding related to cancer? A. Respiratory rate from 24 to 20 breaths/min B. Apical pulse from 80 to 72 beats/min C. Temperature from 98.9° F to 97.9° F D. Blood pressure from 140/90 to 110/70 mm Hg

Blood pressure from 140/90 to 110/70 mm Hg A decrease in blood pressure is the most indicative sign of bleeding. A slight decrease in respiratory rate, apical pulse, and temperature is not the primary indication of bleeding.

When caring for a client with oral cancer who has developed stomatitis as a complication of radiation and chemotherapy, which action does the nurse delegate to the home health aide? A. Provide oral care using disposable foam swabs. B. Inspect the oral mucosa for evidence of oral candidiasis. C. Instruct the client on how to use nystatin (Mycostatin) oral rinses. D. Assist the client in making appropriate dietary choices.

Provide oral care using disposable foam swabs. Providing oral care for a client with oral lesions is an appropriate assignment for a home health aide. Assessments, client teaching, and assisting clients with oral problems in making appropriate dietary choices are the responsibilities of licensed nursing staff.

A client admitted with severe diarrhea is experiencing skin breakdown from frequent stools. What is an important comfort measure for this client? A. Applying hydrocortisone cream B. Cleaning the area with soap and hot water C. Using sitz baths three times daily D. Wearing absorbent cotton underwear

Using sitz baths three times daily Clients with skin breakdown may use sitz baths for comfort 2 or 3 times daily. Barrier creams, not hydrocortisone creams, may be used. The skin should be cleaned gently with soap and warm water. Absorbent cotton underwear helps keep the skin dry, but is not a comfort measure.

A male client with a long history of ulcerative colitis experienced massive bleeding and had emergency surgery for creation of an ileostomy. He is very concerned that sexual intercourse with his wife will be impossible because of his new ileostomy pouch. How does the nurse respond? A. "A change in position may be what is needed for you to have intercourse with your wife." B. "Have you considered going to see a marriage counselor with your wife?" C. "What has your wife said about your pouch system?" D. "You must get clearance from your health care provider before you attempt to have intercourse."

"A change in position may be what is needed for you to have intercourse with your wife." A simple change in positioning during intercourse may alleviate the client's apprehension and facilitate sexual relations with his wife. Suggesting marriage counseling may address the client's concerns, but it focuses on the wrong issue; the client has not stated that he has relationship problems. Asking the client what his wife has said about the pouch may address the client's concerns, but it similarly focuses on the wrong issue. Telling the client that he needs to get clearance from his health care provider is an evasive response that does not address the client's primary concern.

The nurse is teaching a client about dietary choices to prevent dumping syndrome after gastric bypass surgery. Which statement by the client indicates a need for further teaching? A. "I will need to avoid sweetened fruit juice beverages." B. "I can eat ice cream in moderation." C. "I cannot drink alcohol at all." D. "It is okay to have a serving of sugar-free pudding."

"I can eat ice cream in moderation." Milk products such as ice cream must be eliminated from the diet of the client with dumping syndrome. The client with dumping syndrome can no longer consume sweetened drinks. Alcohol must be eliminated from the diet. The client can eat sugar-free pudding, custard, and gelatin with caution.

The nurse is teaching a client how to prevent recurrent chronic gastritis symptoms before discharge. Which statement by the client demonstrates a correct understanding of the nurse's instruction? A. "It is okay to continue to drink coffee in the morning when I get to work." B. "I will need to take vitamin B12 shots for the rest of my life." C. "I should avoid alcohol and tobacco." D. "I should eat small meals about six times a day."

"I should avoid alcohol and tobacco." The client with chronic gastritis should avoid alcohol and tobacco. The client should eliminate caffeine from the diet. The client will need to take vitamin B12 shots only if he or she has pernicious anemia. The client should not eat six small meals daily. This practice may actually stimulate gastric acid secretion.

The RN receives a change-of-shift report about four clients. Which client does the nurse assess first? A. A 20-year-old with ulcerative colitis (UC) who had six liquid stools during the previous shift B. A 25-year-old who has just been admitted with possible appendicitis and has a temperature of 102° F C. A 56-year-old who had a colon resection earlier in the day and whose colostomy bag does not have any stool in it D. A 60-year-old admitted with acute gastroenteritis who is reporting severe cramping and nausea

A 25-year-old who has just been admitted with possible appendicitis and has a temperature of 102° F This client with possible appendicitis may have developed a perforation and may be at risk for peritonitis. Rapid assessment and possible surgical intervention are needed. The client with UC who had six liquid stools, the client whose colostomy bag does not have any stool in it, and the client who was admitted with acute gastroenteritis all need assessment and intervention by an RN, but they are not at immediate risk for life-threatening complications. The client with possible appendicitis has a life-threatening emergency.

A nurse assesses clients at a community health center. Which client is at highest risk for the development of colorectal cancer? a. A 37-year-old who drinks eight cups of coffee daily b. A 44-year-old with irritable bowel syndrome (IBS) c. A 60-year-old lawyer who works 65 hours per week d. A 72-year-old who eats fast food frequently

ANS: D Colon cancer is rare before the age of 40, but its incidence increases rapidly with advancing age. Fast food tends to be high in fat and low in fiber, increasing the risk for colon cancer. Coffee intake, IBS, and a heavy workload do not increase the risk for colon cancer.

The nurse finds a client vomiting coffee-ground emesis. On assessment, the client has blood pressure of 100/74 mm Hg, is acutely confused, and has a weak and thready pulse. Which intervention is the nurse's first priority? A. Administering a histamine2 (H2) antagonist B. Initiating enteral nutrition C. Administering intravenous (IV) fluids D. Administering antianxiety medication

Administering intravenous (IV) fluids Administering IV fluids is necessary to treat the hypovolemia caused by acute gastrointestinal (GI) bleeding. Administration of an H2 antagonist will not treat the basic problem, which is upper GI bleeding. Enteral nutrition will not be part of the treatment plan for acute GI bleeding. Administration of antianxiety medication will not treat the basic problem causing the client's change in mental status, which is hypovolemia.

A client has undergone a partial glossectomy for cancer. What community resource does the nurse refer the client to when dressing supplies will be needed at home? A. Oral Cancer Foundation B. American Cancer Society (ACS) C. Client Advocate Foundation D. American Medical Supply Foundation

American Cancer Society (ACS) The ACS supplies dressings and transportation to and from follow-up visits or medical treatments for clients with cancer. The Oral Cancer Foundation is an organization for local support groups and resources. The Client Advocate Foundation provides education, legal counseling, and referrals to clients with cancer and survivors concerning managed care, insurance, financial issues, job discrimination, and debt crisis matters. The American Medical Supply Foundation does not exist.

A client in the outpatient clinic tells the nurse about experiencing heartburn and nighttime coughing episodes. Which action does the nurse take first? A. Teach the client about antacid effects and side effects. B. Ask the client about medications and dietary intake. C. Suggest that the client sleep with the head elevated 6 inches. D. Tell the client to avoid drinking alcohol late in the evening.

Ask the client about medications and dietary intake. The nurse's initial action should be further assessment of the client's risk factors for gastroesophageal reflux disease. Before suggesting interventions or beginning client teaching, the nurse must elicit more information about the client's symptoms. The nurse needs additional data before telling the client to avoid drinking alcohol late in the evening.

The nurse is caring for a client diagnosed with aphthous ulcers. The nurse instructs the client to avoid which foods? (Select all that apply.) A. Apples B. Bananas C. Cheese D. Nuts E. Potatoes

C. Cheese D. Nuts E. Potatoes Certain foods such as cheese, nuts, and potatoes may trigger allergic responses that cause aphthous ulcers. Apples and bananas are not acidic and do not trigger allergic responses that cause aphthous ulcers.

The nurse is reviewing the medication history for a client diagnosed with gastroesophageal reflux disease who has been prescribed esomeprazole (Nexium) once daily. The client reports that the drug doesn't completely control the symptoms. The nurse contacts the provider to discuss which intervention? A. Adding a second proton pump inhibitor medication B. Increasing the dose of esomeprazole C. Changing to a twice-daily dosing regimen D. Switching to omeprazole (Prilosec)

Changing to a twice-daily dosing regimen The proton pump inhibitors are usually effective when given once daily, but can be given twice daily if symptoms are not well controlled. Adding a second medication, increasing the dose, or switching to another proton pump inhibitor is not recommended.

A client has undergone a radical neck dissection for cancer and is being discharged home while undergoing radiation therapy. Which is likely to be the most important aspect of this client's outpatient care? A. Dental care B. Infection prevention C. Nutrition services D. Support group for cancer survivors

Dental care If radiation therapy is part of the treatment plan, dental care is an important aspect of home care management for these clients. Acute problems include stomatitis and mucositis, and long-term effects include dental decay and xerostomia. Infection prevention, nutrition services, and support groups are part of the overall treatment plan as well, but are not as important.

The admission assessment for a client with acute gastric bleeding indicates blood pressure 82/40 mm Hg, pulse 124 beats/min, and respiratory rate 26 breaths/min. Which admission request does the nurse implement first? A. Type and crossmatch for 4 units of packed red blood cells. B. Infuse lactated Ringer's solution at 200 mL/hr. C. Give pantoprazole (Protonix) 40 mg IV now and then daily. D. Insert a nasogastric tube and connect to low intermittent suction.

Infuse lactated Ringer's solution at 200 mL/hr. The client's most immediate concern is the hypotension associated with volume loss. The most rapidly available volume expanders are crystalloids to treat hypovolemia. A type and crossmatch, administration of pantoprazole, and insertion of a nasogastric tube must all be done, but the nurse's immediate concern is correcting the client's hypovolemia.

A client has developed gastroenteritis while traveling outside the country. What is the likely cause of the client's symptoms? A. Bacteria on the client's hands B. Ingestion of parasites in the water C. Insufficient vaccinations D. Overcooked food

Ingestion of parasites in the water A main cause of gastroenteritis when traveling outside the country is ingestion of water that is infested with parasites. Bacteria on the client's hands will not produce gastroenteritis unless food or water is contaminated with the bacteria. Insufficient vaccinations may cause other disease processes, but not gastroenteritis. Undercooked, not overcooked, food may produce gastroenteritis.

Which of these assigned clients does the nurse assess first after receiving the change-of-shift report? A. Young adult admitted the previous day with abdominal pain who is scheduled for a computed tomography (CT) scan in 30 minutes B. Adult with gastroesophageal reflux disease (GERD) who is describing epigastric pain at a level of 6 (0-to-10 pain scale) C. Middle-aged adult with an esophagogastrectomy done 2 days earlier who has bright-red drainage from the nasogastric (NG) tube D. Older adult admitted with an ileus who has absent bowel sounds and a prescription for metoclopramide (Reglan) on an as-needed (PRN) basis

Middle-aged adult with an esophagogastrectomy done 2 days earlier who has bright-red drainage from the nasogastric (NG) tube The presence of blood in NG drainage is an unexpected finding 2 days after esophagogastrectomy and requires immediate investigation. The young adult scheduled for a CT scan, the adult with GERD, and the older adult with an ileus are all stable and do not require the nurse's immediate attention.

The nurse is caring for an older adult male client who reports stomach pain and heartburn. Which symptom is most significant in determining whether the client's ulceration is gastric or duodenal in origin? A. Pain occurs 1½ to 3 hours after a meal, usually at night. B. Pain is worsened by the ingestion of food. C. The client has a malnourished appearance. D. The client is a man older than 50 years.

Pain occurs 1½ to 3 hours after a meal, usually at night. A key symptom of duodenal ulcers is that pain usually awakens the client between 1:00 a.m. and 2:00 a.m., occurring 1½ to 3 hours after a meal. Pain that is worsened with ingestion of food and a malnourished appearance are key features of gastric ulcers. A male over 50 years is a finding that could apply to either type of ulcer.

Which practice does the nurse include when teaching a client about proper oral care? A. Perform self-examination of the mouth every week, and report any unusual findings. B. Brush the teeth daily and floss as needed. C. Use drugs that reduce the flow of saliva unless lesions are present. D. Getting daily sun exposure is essential to maintain good health.

Perform self-examination of the mouth every week, and report any unusual findings. Clients must be taught to perform self-examination of the mouth every week and to report any unusual findings. Clients should brush teeth and floss every day. Clients should be instructed to avoid, if possible, drugs that can cause inflammation of the mouth or that can reduce the flow of saliva. Clients must be instructed to avoid or limit sun exposure, which is a risk factor for skin cancer.

The nurse is reviewing admitting requests for a client admitted to the intensive care unit with perforation of a duodenal ulcer. Which request does the nurse implement first? A. Apply antiembolism stockings. B. Place a nasogastric (NG) tube, and connect to suction. C. Insert an indwelling catheter, and check output hourly. D. Give famotidine (Pepcid) 20 mg IV every 12 hours.

Place a nasogastric (NG) tube, and connect to suction. To decrease spillage of duodenal contents into the peritoneum, NG suction should be rapidly initiated. This will minimize the risk for peritonitis. Antiembolism stockings will need to be applied, monitoring output is important, and famotidine (Pepcid) will need to be administered, but the nurse's first priority is to minimize the risk for peritonitis.

A client demonstrates the manifestations of diverticulitis with a suspected complication of peritonitis. What is the priority nursing intervention? A. Assessing the client for changes in vital signs B. Medicating the client for pain C. Monitoring for changes in the client's mentation D. Preparing the client for emergency surgery

Preparing the client for emergency surgery The highest priority for this client is to prepare him or her for emergency surgery so that the source of the infection can be removed. It is expected that the client will experience changes in vital signs as a result of the infectious process and accompanying pain. Although monitoring the client's vital signs is important, the client has an immediate need to go to surgery. Medicating the client for pain and determining whether the client is experiencing changes in mentation are important, but are not the highest priority.

A client with an inoperable esophageal tumor is receiving swallowing therapy. Which task does the home health nurse delegate to an experienced home health aide? A. Teaching family members how to determine whether the client is obtaining adequate nutrition B. Assessing lung sounds for possible aspiration when the client is swallowing clear liquids C. Reminding the client to use the chin-tuck technique each time the client attempts to swallow D. Instructing family members about symptoms that may indicate a need to call the provider

Reminding the client to use the chin-tuck technique each time the client attempts to swallow The role of a home health aide when caring for a client with swallowing difficulty includes reinforcement of previously taught swallowing techniques. Client teaching and providing instructions to family members are not within the scope of practice of a home health aide and should be done by the nurse. Likewise, assessment is part of the nursing process and should be done by a nurse.

Which food does the nurse instruct a client undergoing chemotherapy for oral cancer with secondary stomatitis to avoid? A. Broiled fish B. Ice cream C. Salted pretzels D. Scrambled eggs

Salted pretzels Salty foods like pretzels can further irritate ulcers in the client's mouth, causing pain. Foods high in protein, such as fish, eggs, and ice cream, may be included in the diet of the client with stomatitis.

A client has been discharged home after surgery for gastric cancer, and a case manager will follow up with the client. To ensure a smooth transition from the hospital to the home setting, which information provided by the hospital nurse to the case manager is given the highest priority? A. Schedule of the client's follow-up examinations and x-ray assessments B. Information on family members' progress in learning how to perform dressing changes C. Copy of the diet plan prepared for the client by the hospital dietitian D. Detailed account of what occurred during the client's surgical procedure

Schedule of the client's follow-up examinations and x-ray assessments Because recurrence of gastric cancer is common, it will be a priority for the client to have follow-up examinations and x-rays, so that a recurrence can be detected quickly. It may take family members a long time to become proficient at tasks such as dressing changes. Although the case manager should be aware of the diet, family members will likely be preparing the client's daily diet, and they should be provided with this information. It is not necessary for the case manager to have details of the client's surgical procedure unless a significant event has occurred during the procedure.

Which client assessment information is correlated with a diagnosis of chronic gastritis? A. Anorexia, nausea, and vomiting B. Frequent use of corticosteroids C. Hematemesis and anorexia D. Treatment with radiation therapy

Treatment with radiation therapy Treatment with radiation therapy is known to be associated with the development of chronic gastritis. Anorexia, nausea, and vomiting are all symptoms of acute gastritis. Corticosteroid use and hematemesis are also more likely to be signs of acute gastritis.

After change-of-shift report, which client does the nurse plan to assess first? A. Young adult who had a tracheostomy tube removed at the end of the last shift B. Adult who has severe xerostomia associated with radiation therapy C. Middle-aged adult who is describing oral pain after a partial glossectomy D. Older adult who has lost 10 pounds (4.5 kg) secondary to stomatitis

Young adult who had a tracheostomy tube removed at the end of the last shift It is essential to assess the patency of the airway after a tracheostomy tube is removed. The airway could be obstructed by bleeding or swelling at the site or by mechanical objects such as bedding. Severe xerostomia, oral pain after a partial glossectomy, and losing 10 pounds secondary to stomatitis are not life-threatening and do not require immediate assessment or intervention by the nurse.

The nurse working during the day shift on the medical unit has just received report. Which client does the nurse plan to assess first? A. Young adult with epigastric pain, hiccups, and abdominal distention after having a total gastrectomy B. Adult who had a subtotal gastrectomy and is experiencing dizziness and diaphoresis after each meal C. Middle-aged client with gastric cancer who needs to receive omeprazole (Prilosec) before breakfast D. Older adult with advanced gastric cancer who is scheduled to receive combination chemotherapy

Young adult with epigastric pain, hiccups, and abdominal distention after having a total gastrectomy The client with epigastric pain is experiencing symptoms of acute gastric dilation, which can disrupt the suture line. The surgeon should be notified immediately because the nasogastric tube may need irrigation or re-positioning. The client who had a subtotal gastrectomy is not in a life-threatening situation and does not require immediate assessment. The client with gastric cancer and the older adult with advanced gastric cancer are in stable condition and do not require immediate assessment.

A client has been diagnosed with mild gastroesophageal reflux disease and asks the nurse about nonpharmacologic treatments to prevent symptoms. What does the nurse tell this client? A. "Avoid caffeine-containing foods and beverages." B. "Eat three meals each day and avoid snacking between meals." C. "Peppermint lozenges help to reduce stomach upset." D. "Sleep on your left side with a pillow between your knees."

"Avoid caffeine-containing foods and beverages." Teach the client to limit or eliminate foods that decrease lower esophageal sphincter (LES) pressure and that irritate inflamed tissue, causing heartburn, such as peppermint, chocolate, alcohol, fatty foods (especially fried), caffeine, and carbonated beverages. Large meals increase the volume of and pressure in the stomach and delay gastric emptying. Remind the client to eat four to six small meals each day rather than three large ones. Peppermint decreases LES pressure and increases the risk of symptoms. Clients should be taught to elevate the head by 6 to 12 inches for sleep to prevent nighttime reflux.

A nurse is teaching a client with Crohn's disease about managing the disease with the drug adalimumab (Humira). Which instruction does the nurse emphasize to the client? A. "Avoid large crowds and anyone who is sick." B. "Do not take the medication if you are allergic to foods with fatty acids." C. "Expect difficulty with wound healing while you are taking this drug." D. "Monitor your blood pressure and report any significant decrease in it."

"Avoid large crowds and anyone who is sick." The client should avoid being around large crowds to prevent developing an infection. The client should not take the medication if he or she is allergic to certain proteins. Although immune suppression may occur to some degree, the client should not experience difficulty with wound healing while taking adalimumab. The client should not experience a decrease in blood pressure from taking this drug.

A client diagnosed with ulcerative colitis is to be discharged on loperamide (Imodium) for symptomatic management of diarrhea. What does the nurse include in the teaching about this medication? A. "Be aware of the symptoms of toxic megacolon that we discussed." B. "If diarrhea increases, you should let your health care provider know." C. "Pregnancy should be avoided." D. "You will need to decrease your dose of sulfasalazine (Azulfidine)."

"Be aware of the symptoms of toxic megacolon that we discussed." Antidiarrheal drugs may precipitate colonic dilation and toxic megacolon. Toxic megacolon is characterized by an enlarged colon with fever, leukocytosis, and tachycardia. Loperamide will decrease diarrhea rather than increase it. Constipation is sometimes a problem. No contraindication for pregnancy is noted. Sulfasalazine therapy typically continues on a long-term basis.

A certified Wound, Ostomy, and Continence Nurse is teaching a client about caring for a new ileostomy. What information is most important to include? A. "After surgery, output from your ileostomy may be a loose, dark-green liquid with some blood present." B. "Call the health care provider if your stoma has a bluish or pale look." C. "Notify the health care provider if output from your stoma has a sweetish odor." D. "Remember that you must wear a pouch system at all times."

"Call the health care provider if your stoma has a bluish or pale look." If the stoma has a bluish, pale, or dark look, its blood supply may be compromised and the health care provider must be notified immediately. It is true that output from the stoma after surgery may be a loose, greenish-colored liquid that may contain some blood, but this information is not the highest priority for instruction. It is normal for output from the stoma to have very little odor or a sweetish smell. Although it is true that the client will be required to wear a pouch system at all times, this is not the highest priority for instruction.

A nurse is teaching a client about dietary methods to help manage exacerbations ("flare-ups") of diverticulitis. What does the nurse advise the client? A. "Be sure to maintain an exclusively low-fiber diet to prevent pain on defecation." B. "Consume a low-fiber diet while your diverticulitis is active. When inflammation resolves, consume a high-fiber diet." C. "Maintain a high-fiber diet to prevent the development of hemorrhoids that frequently accompany this condition." D. "Make sure you consume a high-fiber diet while diverticulitis is active. When inflammation resolves, consume a low-fiber diet."

"Consume a low-fiber diet while your diverticulitis is active. When inflammation resolves, consume a high-fiber diet." The most effective way to manage diverticulitis is with a low-fiber diet while inflammation is present, followed by a high-fiber diet once the inflammation has subsided. Neither an exclusively low-fiber diet or an exclusively high-fiber diet will effectively manage diverticulitis. A high-fiber diet while diverticulitis is active will only worsen the disease and its symptoms.

A client who developed viral gastroenteritis with vomiting and diarrhea is scheduled to be seen in the clinic the following day. What will the nurse teach the client to do in the meantime? A. "Avoid all solid foods to allow complete bowel rest." B. "Consume extra fluids to replace fluid losses." C. "Take an over-the-counter antidiarrheal medication." D. "Contact your provider for an antibiotic medication."

"Consume extra fluids to replace fluid losses." Clients should be taught to drink extra fluids to replace fluid lost through vomiting and diarrhea. It is not necessary to stop all solid food intake. Antidiarrheal medications are used if diarrhea is severe. Antibiotics are used if the infection is bacterial.

A client who has been diagnosed recently with esophageal cancer states, "I'm not comfortable going to my father's birthday lunch at our family-owned restaurant because I'm afraid I'll choke in public." What is the nurse's best response? A. "I understand your concerns, but you can't give up your normal activities. You should go anyway and try not to worry about it." B. "Could you perhaps invite everyone over to cook at your home? That will allow you to be together and be more relaxed." C. "Why not take one of your antianxiety pills before going? That will keep you from worrying about everything so much." D. "You need to talk to your doctor about your concerns. The doctor may recommend that you join a support group for cancer survivors."

"Could you perhaps invite everyone over to cook at your home? That will allow you to be together and be more relaxed." Suggesting that the client invite people over for a meal provides psychosocial support to the client and assists the client in finding a solution to the problem. Telling the client not to worry about it or to call the provider is evasive and unhelpful; it is used to placate the client and does not address the client's concerns. The client should use problem-solving and coping skills before resorting to the use of medication.

27. The nurse knows that the ideal time to change an ostomy pouch is a. Before eating a meal, when the patient is comfortable. b. When the patient feels that he needs to have a bowel movement. c. When ordered in the patient's chart. d. After the patient has ambulated the length of the hallway.

ANS: A The nurse wants to change the ostomy appliance when as little output as necessary ensures a smooth procedure. Patients with ostomies do not feel the urge to defecate because the sensory portion of the anus is not stimulated. Changing the ostomy pouch is a nursing judgment decision. After a patient ambulates, stool output is increased.

A client with peptic ulcer disease asks the nurse whether a maternal history of gastric cancer will cause the client to develop gastric cancer. What is the nurse's best response? A. "Yes, it is known that a family history of gastric cancer will cause someone to develop gastric cancer." B. "If you are concerned that you are at high risk, I recommend speaking to your provider about the possibility of genetic testing." C. "Have you spoken to your health care provider about your concerns?" D. "I wouldn't be too concerned about that as long as your diet limits pickled, salted, and processed food."

"If you are concerned that you are at high risk, I recommend speaking to your provider about the possibility of genetic testing." Genetic counseling will help the client determine whether he or she is at exceptionally high risk to develop gastric cancer. The client cannot know for certain whether family history places him or her at exceptionally high risk to develop gastric cancer unless specific testing is done. Asking the client what the provider has said is an evasive answer by the nurse and does not help answer the client's question. Although a diet high in pickled, salted, and processed foods does increase the risk for gastric cancer, a family history of specific types of cancer can also increase the risk.

A client with an exacerbation of ulcerative colitis has been prescribed Vivonex PLUS. The client asks the nurse how this is helpful for improving symptoms. How does the nurse reply? A. "It is absorbed quickly and allows the affected part of the GI tract to rest and heal." B. "It provides key nutrients and extra calories to promote healing." C. "It is bland and reduces the secretion of gastric acids." D. "It does not contain caffeine or other GI tract stimulants."

"It is absorbed quickly and allows the affected part of the GI tract to rest and heal." For less severe exacerbations, an elemental or semi-elemental product such as Vivonex PLUS may be prescribed to induce remission. These products are absorbed in the jejunum and therefore permit the distal small intestine and colon to rest. Nutritional supplements such as Ensure or Sustacal are added to provide nutrients and more calories. GI stimulants such as caffeinated beverages and alcohol should be avoided, but this is not the reason for using Vivonex PLUS.

A client with ulcerative colitis (UC) has stage 1 of a restorative proctocolectomy with ileo-anal anastomosis (RPC-IPAA) procedure performed. The client asks the nurse, "How long do people with this procedure usually have a temporary ileostomy?" How does the nurse respond? A. "It is usually ready to be closed in about 1 to 2 months." B. "This is something that you will have to discuss with your health care provider." C. "The period of time is indefinite—I am sorry that I cannot say." D. "You will probably have it for 6 months or longer, until things heal.

"It is usually ready to be closed in about 1 to 2 months." The RPC-IPAA has become the most effective alternate method for UC clients who have surgery to remove diseased portions of intestines. Stage 1 creates a temporary ileostomy to be used while an internally created pouch is healing. Stage 2 closes the ileostomy, and the client begins to use the pouch for storage of stool. The time between the surgeries is generally 1 to 2 months. Telling the client that he or she will have to discuss it with the health care provider evades the question; the nurse can give generalities to the client based on past practice and available data. The time that the client has the ileostomy is not "indefinite." The intent of this procedure is to eliminate the need to have a permanent ileostomy. The pouch should heal in 1 to 2 months, not 6 months; this estimate is not based on the expected outcome.

A client has been newly diagnosed with ulcerative colitis (UC). What does the nurse teach the client about diet and lifestyle choices? A. "Drinking carbonated beverages will help with your abdominal distress." B. "It's OK to smoke cigarettes, but you should limit them to ½ pack per day." C. "Lactose-containing foods should be reduced or eliminated from your diet." D. "Raw vegetables and high-fiber foods may help to diminish your symptoms."

"Lactose-containing foods should be reduced or eliminated from your diet." Lactose-containing foods are often poorly tolerated and should be reduced or eliminated from the diet of clients with UC. Carbonated beverages are GI stimulants that can cause discomfort and should be used rarely or completely eliminated from the diet. Cigarette smoking is a stimulant that can cause GI distress symptoms; nurses should never advise clients that any amount of cigarette smoking is "OK." Raw vegetables and high-fiber foods can cause GI symptoms in clients with UC.

The nurse is teaching a client with peptic ulcer disease about the prescribed drug regimen. Which statement made by the client indicates a need for further teaching before discharge? A. "Nizatidine (Axid) needs to be taken three times a day to be effective." B. "Taking ranitidine (Zantac) at bedtime should decrease acid production at night." C. "Sucralfate (Carafate) should be taken 1 hour before and 2 hours after meals." D. "Omeprazole (Prilosec) should be swallowed whole and not crushed."

"Nizatidine (Axid) needs to be taken three times a day to be effective." Nizatidine is most effective if administered once daily. A dose of ranitidine at bedtime should decrease acid production throughout the night. Sucralfate should be taken 1 hour before a

A client has been diagnosed with terminal gastric cancer and is interested in obtaining support from hospice, but expresses concern that pain management will not be adequate. What is the nurse's best response? A. "Pain control is a major component of the care provided by hospice and its staff members." B. "What has your provider told you about participating in hospice?" C. "I can speak to your provider about requesting adequate pain medication." D. "You don't want to become too dependent on pain medication and become an addict."

"Pain control is a major component of the care provided by hospice and its staff members." Telling the client that pain control is a major component of hospice care correctly describes the services provided by hospice and its staff members, and reassures the client about their expertise in pain management. Asking the client what the provider has said is an evasive response by the nurse and does not address the client's concerns. The nurse does not need to speak to the provider because pain control is an integral part of hospice services. It is inappropriate to tell a terminally ill client in need of pain control that he or she may become too dependent on pain medication.

A client with gastroesophageal reflux disease has undergone a laparoscopic Nissen fundoplication (LNF). What will the nurse include in postoperative home care instructions? A. "Consume carbonated beverages if you experience stomach upset." B. "Remain on a soft diet for about a week and avoid raw fruits and vegetables." C. "You may resume running and weight lifting if you wish." D. "You may stop taking your anti-reflux medications after 1 week."

"Remain on a soft diet for about a week and avoid raw fruits and vegetables." After LNF, clients should be taught to remain on a soft diet for 1 week. Carbonated beverages should be avoided. Clients may walk, but should avoid heavy lifting. Anti-reflux medications should be taken for 1 month after the procedure.

The nurse has placed a nasogastric (NG) tube in a client with upper gastrointestinal (GI) bleeding to administer gastric lavage. The client asks the nurse about the purpose of the NG tube for the procedure. What is the nurse's best response? A. "Saline goes down the tube to help clean out your stomach." B. "Medication goes down the tube to help clean out your stomach." C. "The provider requested the tube to be placed just in case it was needed." D. "We'll start feeding you through it once your stomach is cleaned out."

"Saline goes down the tube to help clean out your stomach." Gastric lavage involves the instillation of water or saline through an NG tube to clear out stomach contents and blood clots. It does not involve the instillation of medication. An NG tube is not typically placed in a client without a particular purpose in mind. Gastric lavage does not involve enteral feeding.

A client with peptic ulcer disease (PUD) asks the nurse whether licorice and slippery elm might be useful in managing the disease. What is the nurse's best response? A. "No, they probably won't be useful. You should use only prescription medications in your treatment plan." B. "These herbs could be helpful. However, you should talk with your provider before adding them to your treatment regimen." C. "Yes, these are known to be effective in managing this disease, but make sure you research the herbs thoroughly before taking them." D. "No, herbs are not useful for managing this disease. You can use any type of over-the-counter drugs though. They have been shown to be safe."

"These herbs could be helpful. However, you should talk with your provider before adding them to your treatment regimen." Although licorice and slippery elm may be helpful in managing PUD, the client should consult his or her health care provider before making a change in the treatment regimen. Alternative therapies may or may not be helpful in managing PUD. The client should not use over-the-counter medications without first discussing it with his or her provider.

An older female client is diagnosed with gastric cancer. Which statement made by the client's family demonstrates a correct understanding of the disorder? A. "This may be related to her recurring ulcer disease." B. "This is probably curable with surgery." C. "Gastric cancer has a strong genetic component." D. "Thank goodness she won't have to undergo surgery."

"This may be related to her recurring ulcer disease." Infection with Helicobacter pylori is the largest risk factor for gastric cancer because it carries the cytotoxin-associated antigen A (CagA) gene. Clients with chronic ulcers are probably infected with this organism. Surgery is not curative; most gastric cancers do not present with symptoms until late in the disease and have a high fatality rate. There is no strong genetic predisposition to gastric cancer. Surgery is part of the treatment.

A client has a long-term history of Crohn's disease and has recently developed acute gastritis. The client asks the nurse whether Crohn's disease was a direct cause of the gastritis. What is the nurse's best response? A. "Yes, Crohn's disease is known to be a direct cause of the development of chronic gastritis." B. "We know that there can be an association between Crohn's disease and chronic gastritis, but Crohn's does not directly cause acute gastritis to develop." C. "What has your doctor told you about how your gastritis developed?" D. "Yes, a familial tendency to inherit Crohn's disease and gastritis has been reported. Have your other family members been tested for Crohn's disease?"

"We know that there can be an association between Crohn's disease and chronic gastritis, but Crohn's does not directly cause acute gastritis to develop." Crohn's disease may be an underlying disease process when chronic gastritis develops, but not when acute gastritis occurs. It is not known to be a direct cause of the disease. Although Crohn's disease tends to run in families, gastritis is a symptom of other disease processes and is not a disease process in and of itself. Asking the client what the doctor has said is an evasive response on the part of the nurse and does not help answer the client's question.

A client has been diagnosed with terminal esophageal cancer. The client is interested in obtaining support from hospice, but expresses concern that pain management will not be adequate. What is the nurse's best response? A. "Haven't you received adequate pain management in the hospital?" B. "Would you like me to get a nurse from hospice to come talk with you?" C. "Do you want me to call the hospital chaplain to explain hospice to you?" D. "Talk to your health care provider about hospice services."

"Would you like me to get a nurse from hospice to come talk with you?" The best way to alleviate the client's concerns would be to have a hospice nurse talk with the client and answer any questions. Suggesting that the client has had adequate pain management sounds defensive. Referring the client to the chaplain or the health care provider is evasive and attempts to shift responsibility away from the nurse.

A client is admitted with severe viral gastroenteritis caused by norovirus. The client asks the nurse, "How did I get this disease?" Which answer by the nurse is correct? A. "You may have contracted it from an infected infant." B. "You may have consumed contaminated food or water." C. "You may have come into contact with an infected animal." D. "You may have had contact with the blood of an infected person."

"You may have consumed contaminated food or water." Norovirus is the leading foodborne disease that causes gastroenteritis. It is transmitted via the fecal-oral route from person to person and from contaminated food and water. Vomiting causes the virus to become airborne. Campylobacter can be transmitted by contact with infected infants or animals. Escherichia coli may be spread via animals and contaminated food, water, or fomites. HIV may be spread via the blood, but not norovirus. Campylobacter and E. coli both cause bacterial gastroenteritis, while norovirus causes viral gastroenteritis.

A client newly diagnosed with ulcerative colitis (UC) is started on sulfasalazine (Azulfidine). What does the nurse tell the client about why this therapy has been prescribed? A. "It is to stop the diarrhea and bloody stools." B. "This will minimize your GI discomfort." C. "With this medication, your cramping will be relieved." D. "Your intestinal inflammation will be reduced."

"Your intestinal inflammation will be reduced." Sulfasalazine (Azulfidine) is one of the primary treatments for UC. It is thought to inhibit prostaglandin synthesis and thereby reduce inflammation. Although it is hoped that reduction of inflammation will cause the diarrhea and bloody stools to stop, this is not the way that the drug works. Antidiarrheal drugs "stop" diarrhea. The drug's action as an anti-inflammatory will diminish the client's pain as the inflammation subsides, but this is not the purpose of the drug—it is not an analgesic.

37. A patient has constipation and hypernatremia. The nurse prepares to administer which type of enema? a. Oil retention b. Carminative c. Saline d. Tap water

ANS: D Tap water enema would draw fluid into the system and would help flush out excess sodium. Oil retention would not address sodium problems. Carminative enemas are used to provide relief from distention caused by gas. A saline enema would worsen hypernatremia.

As a result of being treated with radiation for oral cancer, a client is experiencing xerostomia. What community resource does the nurse suggest for this client's care? A. Dentist B. Occupational therapist C. Psychiatrist D. Speech therapist

Dentist Xerostomia is a long-term effect of radiation therapy and requires ongoing oral care such as the use of saliva substitutes and follow-up dental visits. Occupational therapists, psychiatrists, and speech therapists are not the appropriate resource for a client with xerostomia.

Which client does the charge nurse assign to an experienced LPN/LVN? A. A 28-year-old who requires teaching about how to catheterize a Kock ileostomy B. A 30-year-old who must receive neomycin sulfate (Mycifradin) before a colectomy C. A 34-year-old with ulcerative colitis (UC) who has a white blood cell count of 23,000/mm3 D. A 38-year-old with gastroenteritis who is receiving IV fluids at 250 mL/hr

A 30-year-old who must receive neomycin sulfate (Mycifradin) before a colectomy The LPN/LVN should be familiar with the purpose, adverse effects, and client teaching required for neomycin. Teaching about how to catheterize a Kock ileostomy, assessing the client with UC with a high white blood cell count, and monitoring the client with gastroenteritis receiving IV fluids present complex problems that require assessment or intervention by an RN.

An intensive care unit (ICU) RN is "floated" to the medical-surgical unit. Which client does the charge nurse assign to the float nurse? A. A 28-year-old with an exacerbation of Crohn's disease (CD) who has a draining enterocutaneous fistula B. A 32-year-old with ulcerative colitis (UC) who needs discharge teaching about the use of hydrocortisone enemas C. A 34-year-old who has questions about how to care for a newly created ileo-anal reservoir D. A 36-year-old with peritonitis who just returned from surgery with multiple drains in place

A 36-year-old with peritonitis who just returned from surgery with multiple drains in place The ICU nurse is familiar with the care of a client with peritonitis, including monitoring for complications such as sepsis and kidney failure. The client with CD who has a draining enterocutaneous fistula, the client with UC who needs discharge teaching, and the client with questions about an ileo-anal reservoir are best assigned to a medical-surgical nurse who is more familiar with the care and teaching needed for clients with their respective disorders.

The nurse is instructing a client with recently diagnosed diverticular disease about diet. What food does the nurse suggest the client include? A. A slice of 5-grain bread B. Chuck steak patty (6 ounces) C. Strawberries (1 cup) D. Tomato (1 medium)

A slice of 5-grain bread Whole-grain breads are recommended to be included in the diet of clients with diverticular disease because cellulose and hemicellulose types of fiber are found in them. Dietary fat should be reduced in clients with diverticular disease. If the client wants to eat beef, it should be of a leaner cut. Foods containing seeds, such as strawberries, should be avoided. Tomatoes should be avoided unless the seeds are removed. The seeds may block diverticula in the client and present problems leading to diverticulitis.

The nurse is teaching a client who recently began taking sulfasalazine (Azulfidine) about the drug. What side effects does the nurse tell the client to report to the health care provider? (Select all that apply.) A. Anorexia B. Depression C. Drowsiness D. Frequent urination E. Headache F. Vomiting

A. Anorexia E. Headache F. Vomiting Anorexia, headache, and nausea/vomiting are side effects of sulfasalazine that should be reported to the health care provider. Depression, drowsiness, and urinary problems are not side effects of sulfasalazine.

The nurse is instructing a client on measures to maintain effective oral health. Which measures does the nurse include in the client's teaching plan? (Select all that apply.) A. Regular dental checkups B. Use of mouthwashes containing alcohol C. Ensuring that dentures are slightly loose-fitting D. Managing stress as much as possible E. Eating a balanced diet

A. Regular dental checkups D. Managing stress as much as possible E. Eating a balanced diet Regular dental checkups are important so potential problems can be prevented or attended to promptly. Stress suppresses the immune system, which can increase the client's risk for infections such as Candida albicans. Eating a balanced diet can reduce the risk for dental caries and infections such as C. albicans or stomatitis. Mouthwashes that contain alcohol may cause inflammation and should be avoided. Dentures should be in good repair and should fit properly.

24. A patient is diagnosed with a bowel obstruction. The nurse chooses which type of tube for gastric decompression? a. Salem sump b. Dobhoff c. Sengstaken-Blakemore d. Small bore

ANS: A A bowel obstruction causes a backup into the gastric area; a nasogastric tube may be inserted to decompress secretions and gases from the gastrointestinal tract. The Salem sump has the width and functionality needed to both feed and suction, and it is ideal for a bowel obstruction. A Dobhoff tube is used for instillation of feedings. A Sengstaken-Blakemore tube is used to compress stomach contents to prevent hemorrhage. A small bore is intended for nutritional feedings only and does not have suction capacity.

5. A nurse is assisting a patient in making dietary choices that promote healthy bowel elimination. Which menu option should the nurse recommend? a. Grape and walnut chicken salad sandwich on whole wheat bread b. Broccoli and cheese soup with potato bread c. Dinner salad topped with hard-boiled eggs, cheese, and fat-free dressing d. Turkey and mashed potatoes with brown gravy

ANS: A A healthy diet for the bowel should include foods high in bulk-forming fiber. Whole grains, fresh fruit, and fresh vegetables are excellent sources. Foods without much fiber and with high levels of fat can slow down peristalsis, causing constipation.

A nurse assesses a client who is prescribed 5-fluorouracil (5-FU) chemotherapy intravenously for the treatment of colon cancer. Which assessment finding should alert the nurse to contact the health care provider? a. White blood cell (WBC) count of 1500/mm3 b. Fatigue c. Nausea and diarrhea d. Mucositis and oral ulcers

ANS: A Common side effects of 5-FU include fatigue, leukopenia, diarrhea, mucositis and mouth ulcers, and peripheral neuropathy. However, the clients WBC count is very low (normal range is 5000 to 10,000/mm3), so the provider should be notified. He or she may want to delay chemotherapy by a day or two. Certainly the client is at high risk for infection. The other assessment findings are consistent with common side effects of 5-FU that would not need to be reported immediately.

23. A nurse is educating a patient on how to irrigate an ostomy bag. Which statement by the patient indicates the need for further instruction? a. "I can use a fleet enema to save money because it contains the same irrigation solution." b. "Sitting on the toilet lets the irrigation sleeve eliminate into the bowl." c. "I should never attempt to reach into my stoma to remove fecal material." d. "Using warm tap water will reduce cramping and discomfort during the procedure."

ANS: A Enema applicators should never be used in the stoma because they can cause damage. A special coned irrigation device is used for ostomies. Irrigating a stoma into the toilet is an effective management technique. Fingers and other objects should not be placed into the stoma because they may cause trauma to the intestinal wall. Warm tap water will reduce cramping during irrigation.

6. A patient informs the nurse that she was using laxatives three times daily to lose weight. After stopping use of the laxative, the patient had difficulty with constipation and wonders if she needs to take laxatives again. The nurse educates the patient that a. Long-term laxative use causes the bowel to become less responsive to stimuli, and constipation may occur. b. Laxatives can cause trauma to the intestinal lining and scarring may result, leading to decreased peristalsis. c. Natural laxatives such as mineral oil are safer than chemical laxatives for relieving constipation. d. Laxatives cause the body to become malnourished, so when the patient begins eating again, the body absorbs all of the food, and no waste products are produced.

ANS: A Long-term laxative use can lead to constipation. Increasing fluid and fiber intake can help with this problem. Laxatives do not cause scarring. Natural laxatives like mineral oil come with their own set of risks, such as inability to absorb fat-soluble vitamins. Even if malnourished, the body will produce waste if substance is consumed.

22. While a cleansing enema is administered to an 80-year-old patient, the patient expresses the urge to defecate. What is the next priority nursing action? a. Positioning the patient in the dorsal recumbent position with a bed pan b. Assisting the patient to the bedside commode c. Stopping the enema cleansing and rolling the patient into right-lying Sims' position d. Inserting a rectal plug to contain the enema solution

ANS: A Patients with poor sphincter control may not be able to hold in all of an enema solution. Positioning the patient on a bed pan in dorsal recumbent position will allow the nurse to continue to administer the enema. Having the patient get up to toilet is unsafe because the rectal tube can damage the mucosal lining. The purpose of the enema is to promote defecation; stopping it early may inhibit its effectiveness. Use of a rectal plug to contain the solution is inappropriate.

39. The nurse should place the patient in which position when preparing to administer an enema? a. Left Sims' position b. Fowler's c. Supine d. Semi-Fowler's

ANS: A Side-lying Sims' position allows the enema solution to flow downward by gravity along the natural curve of the sigmoid colon. This helps to improve retention of the enema. Administering an enema in a sitting position may allow the curved rectal tube to scrape the rectal wall.

After teaching a client who has a femoral hernia, the nurse assesses the clients understanding. Which statement indicates the client needs additional teaching related to the proper use of a truss? a. I will put on the truss before I go to bed each night. b. Ill put some powder under the truss to avoid skin irritation. c. The truss will help my hernia because I cant have surgery. d. If I have abdominal pain, Ill let my health care provider know right away.

ANS: A The client should be instructed to apply the truss before arising, not before going to bed at night. The other statements show an accurate understanding of using a truss.

40. The nurse is assessing a patient 2 hours after a colonoscopy. Based on the procedure done, what focused assessment will the nurse include? a. Bowel sounds b. Presence of flatulence c. Bowel movements d. Nausea

ANS: A The nurse does want to hear the presence of bowel sounds; absent bowel sounds may indicate a complication from the surgery. Bowel movements and flatulence are not expected in the hours after surgery. The nurse does want to hear the presence of bowel movements. Nausea is not a problem following colonoscopy.

A nurse cares for a client who states, My husband is repulsed by my colostomy and refuses to be intimate with me. How should the nurse respond? a. Lets talk to the ostomy nurse to help you and your husband work through this. b. You could try to wear longer lingerie that will better hide the ostomy appliance. c. You should empty the pouch first so it will be less noticeable for your husband. d. If you are not careful, you can hurt the stoma if you engage in sexual activity. .

ANS: A The nurse should collaborate with the ostomy nurse to help the client and her husband work through intimacy issues. The nurse should not minimize the clients concern about her husband with ways to hide the ostomy. The client will not hurt the stoma by engaging in sexual activity

A nurse cares for a client who has a new colostomy. Which action should the nurse take? a. Empty the pouch frequently to remove excess gas collection. b. Change the ostomy pouch and wafer every morning. c. Allow the pouch to completely fill with stool prior to emptying it. d. Use surgical tape to secure the pouch and prevent leakage.

ANS: A The nurse should empty the new ostomy pouch frequently because of excess gas collection, and empty the pouch when it is one-third to one-half full of stool. The ostomy pouch does not need to be changed every morning. Ostomy wafers with paste should be used to secure and seal the ostomy appliance; surgical tape should not be used.

14. The nurse is devising a plan of care for a patient with the nursing diagnosis of Constipation related to opioid use. Which of the following outcomes would the nurse evaluate as successful for the patient to establish normal defecation? a. The patient reports eliminating a soft, formed stool. b. The patient has quit taking opioid pain medication. c. The patient's lower left quadrant is tender to the touch. d. The nurse hears bowel sounds present in all four quadrants.

ANS: A The nurse's goal is for the patient to be on opioid medication and to have normal bowel elimination. Normal stools are soft and formed. Ceasing pain medication is not a desired outcome for the patient. Tenderness in the left lower quadrant indicates constipation and does not further address bowel elimination. Present bowel sounds indicate that the bowels are moving; however, they are not an indication of defecation.

A nurse cares for a client who had a colostomy placed in the ascending colon 2 weeks ago. The client states, The stool in my pouch is still liquid. How should the nurse respond? a. The stool will always be liquid with this type of colostomy. b. Eating additional fiber will bulk up your stool and decrease diarrhea. c. Your stool will become firmer over the next couple of weeks. d. This is abnormal. I will contact your health care provider.

ANS: A The stool from an ascending colostomy can be expected to remain liquid because little large bowel is available to reabsorb the liquid from the stool. This finding is not abnormal. Liquid stool from an ascending colostomy will not become firmer with the addition of fiber to the clients diet or with the passage of time.

30. Which nursing action best reduces risk of excoriation to the mucosal lining of the nose from a nasogastric tube? a. Lubricating the nares with water-soluble lubricant b. Applying a small ice bag to the nose for 5 minutes every 4 hours c. Instilling Xylocaine into the nares once a shift d. Changing the tape holding the tube in place once a shift

ANS: A The tube constantly irritates the nasal mucosa, increasing the risk of excoriation. Frequent lubrication with a water-soluble lubricant decreases the likelihood of excoriation. Ice is not applied to the nose. Ice may be applied externally to the throat if the patient reports a sore throat. Xylocaine requires a physician order and is used to treat sore throat, not nasal mucosal excoriation. Changing the tape should be done daily, not every shift.

34. A nurse is performing an assessment on a patient who has not had a bowel movement in 3 days. The nurse would expect which other assessment finding? a. Hypoactive bowel sounds b. Jaundice in sclera c. Decreased skin turgor d. Soft tender abdomen

ANS: A Three or more days with no bowel movement indicates hypomotility of the GI tract. Assessment findings would include hypoactive bowel sounds, a firm distended abdomen, and pain or discomfort upon palpation.

A nurse plans care for a client who is recovering from an inguinal hernia repair. Which interventions should the nurse include in this clients plan of care? (Select all that apply.) a. Encouraging ambulation three times a day b. Encouraging normal urination c. Encouraging deep breathing and coughing d. Providing ice bags and scrotal support e. Forcibly reducing the hernia

ANS: A, B, D Postoperative care for clients with an inguinal hernia includes all general postoperative care except coughing. The nurse should promote lung expansion by encouraging deep breathing and ambulation. The nurse should encourage normal urination, including allowing the client to stand, and should provide scrotal support and ice bags to prevent swelling. A hernia should never be forcibly reduced, and this procedure is not part of postoperative care.

A nurse plans care for a client who has chronic diarrhea. Which actions should the nurse include in this clients plan of care? (Select all that apply.) a. Using premoistened disposable wipes for perineal care b. Turning the client from right to left every 2 hours c. Using an antibacterial soap to clean after each stool d. Applying a barrier cream to the skin after cleaning e. Keeping broken skin areas open to air to promote healing

ANS: A, B, D The nurse should use premoistened disposable wipes instead of toilet paper for perineal care, or mild soap and warm water after each stool. Antibacterial soap would be too abrasive and damage good bacteria on the skin. The nurse should apply a thin layer of a medicated protective barrier after cleaning the skin. The client should be re-positioned frequently so that he or she is kept off the affected area, and open skin areas should be covered with DuoDerm or Tegaderm occlusive dressing to promote rapid healing.

A nurse cares for a client who has been diagnosed with a small bowel obstruction. Which assessment findings should the nurse correlate with this diagnosis? (Select all that apply.) a. Serum potassium of 2.8 mEq/L b. Loss of 15 pounds without dieting c. Abdominal pain in upper quadrants d. Low-pitched bowel sounds e. Serum sodium of 121 mEq/L

ANS: A, C, E Small bowel obstructions often lead to severe fluid and electrolyte imbalances. The client is hypokalemic (normal range is 3.5 to 5.0 mEq/L) and hyponatremic (normal range is 136 to 145 mEq/L). Abdominal pain across the upper quadrants is associated with small bowel obstruction. Dramatic weight loss without dieting followed by bowel obstruction leads to the probable development of colon cancer. High-pitched sounds may be noted with small bowel obstructions.

A nurse inserts a nasogastric (NG) tube for an adult client who has a bowel obstruction. Which actions does the nurse perform correctly? (Select all that apply.) a. Performs hand hygiene and positions the client in high-Fowlers position, with pillows behind the head and shoulders b. Instructs the client to extend the neck against the pillow once the NG tube has reached the oropharynx c. Checks for correct placement by checking the pH of the fluid aspirated from the tube d. Secures the NG tube by taping it to the clients nose and pinning the end to the pillowcase e. Connects the NG tube to intermittent medium suction with an anti-reflux valve on the air vent

ANS: A, C, E The clients head should be flexed forward once the NG tube has reached the oropharynx. The NG tube should be secured to the clients gown, not to the pillowcase, because it could become dislodged easily. All the other actions are appropriate.

A nurse cares for a client who has a nasogastric (NG) tube. Which actions should the nurse take? (Select all that apply.) a. Assess for proper placement of the tube every 4 hours. b. Flush the tube with water every hour to ensure patency. c. Secure the NG tube to the clients upper lip. d. Disconnect suction when auscultating bowel peristalsis. e. Monitor the clients skin around the tube site for irritation.

ANS: A, D, E The nurse should assess for proper placement, tube patency, and output every 4 hours. The nurse should also monitor the skin around the tube for irritation and secure the tube to the clients nose. When auscultating bowel sounds for peristalsis, the nurse should disconnect suction.

A nurse assesses a client who is recovering from a hemorrhoidectomy that was done the day before. The nurse notes that the client has lower abdominal distention accompanied by dullness to percussion over the distended area. Which action should the nurse take? a. Assess the clients heart rate and blood pressure. b. Determine when the client last voided. c. Ask if the client is experiencing flatus. d. Auscultate all quadrants of the clients abdomen.

ANS: B Assessment findings indicate that the client may have an over-full bladder. In the immediate postoperative period, the client may experience difficulty voiding due to urinary retention. The nurse should assess when the client last voided. The clients vital signs may be checked after the nurse determines the clients last void. Asking about flatus and auscultating bowel sounds are not related to a hemorrhoidectomy.

15. The nurse is emptying an ileostomy pouch for a patient. Which assessment finding would the nurse report immediately? a. Liquid consistency of stool b. Presence of blood in the stool c. Noxious odor from the stool d. Continuous output from the stoma

ANS: B Blood in the stool may indicate a problem with the surgical procedure, and the physician should be notified. All other options are expected findings for an ileostomy.

9. A patient expresses concerns over having black stool. The fecal occult test is negative. Which response by the nurse is most appropriate? a. "This is probably a false negative; we should rerun the test." b. "Do you take iron supplements?" c. "You should schedule a colonoscopy as soon as possible." d. "Sometimes severe stress can alter stool color."

ANS: B Certain medications and supplements, such as iron, can alter the color of stool. The fecal occult test takes three separate samples over a period of time and is a fairly reliable test. A colonoscopy is health prevention screening that should be done every 5 to 10 years; it is not the nurse's initial priority. Stress alters GI motility and stool consistency, not color.

After teaching a client with irritable bowel syndrome (IBS), a nurse assesses the clients understanding. Which menu selection indicates that the client correctly understands the dietary teaching? a. Ham sandwich on white bread, cup of applesauce, glass of diet cola b. Broiled chicken with brown rice, steamed broccoli, glass of apple juice c. Grilled cheese sandwich, small banana, cup of hot tea with lemon d. Baked tilapia, fresh green beans, cup of coffee with low-fat milk

ANS: B Clients with IBS are advised to eat a high-fiber diet (30 to 40 g/day), with 8 to 10 cups of liquid daily. Chicken with brown rice, broccoli, and apple juice has the highest fiber content. They should avoid alcohol, caffeine, and other gastric irritants.

A nurse cares for a middle-aged male client who has irritable bowel syndrome (IBS). The client states, I have changed my diet and take bulk-forming laxatives, but my symptoms have not gotten better. I heard about a drug called Amitiza. Do you think it might help? How should the nurse respond? a. This drug is still in the research phase and is not available for public use yet. b. Unfortunately, lubiprostone is approved only for use in women. c. Lubiprostone works well. I will recommend this prescription to your provider. d. This drug should not be used with bulk-forming laxatives.

ANS: B Lubiprostone (Amitiza) is a new drug for IBS with constipation that works by simulating receptors in the intestines to increase fluid and promote bowel transit time. Lubiprostone is currently approved only for use in women. Trials with increased numbers of male participants are needed prior to Food and Drug Administration approval for men.

20. A nurse is preparing a patient for a magnetic resonance imaging scan. Which nursing action is most important? a. Ensuring that the patient does not eat or drink 2 hours before the examination b. Removing all of the patient's metallic jewelry c. Administering a colon cleansing product 12 hours before the examination d. Obtaining an order for a pain medication before the test is performed

ANS: B No jewelry or metal products should be in the same room as an MRI machine because of the high-power magnet used in the machine. The patient needs to be NPO 4 to 6 hours before the examination. Colon cleansing products are not necessary for MRIs. Pain medication is not needed before the examination is performed.

4. The nurse is caring for a patient who is confined to the bed. The nurse asks the patient if he needs to have a bowel movement 30 minutes after eating a meal because a. The digested food needs to make room for recently ingested food. b. Mastication triggers the digestive system to begin peristalsis. c. The smell of bowel elimination in the room would deter the patient from eating. d. More ancillary staff members are available after meal times.

ANS: B Peristalsis occurs only a few times a day; the strongest peristaltic waves are triggered by mastication of the meal. The intestine can hold a great deal of food. A patient's voiding schedule should not be based on the staff's convenience.

29. An older adult's perineal skin appears to be dry and thin with mild excoriation. When providing hygiene after a bowel movement, the nurse should a. Thoroughly scrub the skin with a wash cloth and hypoallergenic soap. b. Apply a skin protective lotion after perineal care. c. Tape an occlusive moisture barrier pad to the patient's skin. d. Massage the skin with deep kneading pressure.

ANS: B Proper skin care and perineal cleaning require that the nurse gently clean the skin and apply a moistening barrier cream. Tape and occlusive dressings can damage skin. Excessive pressure and force are inappropriate and may cause skin breakdown.

A client is newly diagnosed with tongue and esophageal cancer. Which response to the diagnosis does the nurse expect the client to have? A. Anxiety from knowing that, as a result of cancer and surgery, ingestion of food by mouth might become impossible B. Concern about getting an infection caused by invasive procedures C. Fear about the chance of aspiration after surgery D. Depression about changes in the face and neck after surgery

Depression about changes in the face and neck after surgery The client will likely be depressed because of expected alterations to the body caused by cancer and probable surgery. Anxiety about nutritional intake, concern about infection, and fear of aspiration are not the client's likely initial responses to a new diagnosis of tongue and esophageal cancer.

26. A nurse is pouching an ostomy on a patient with an ileostomy. Which action by the nurse is most appropriate? a. Changing the skin barrier portion of the ostomy pouch daily b. Selecting a pouch that is able to hold excess output to reduce the frequency of pouch emptying c. Thoroughly scrubbing the skin around the stoma to remove excess stool and adhesive d. Measuring the correct size for the barrier device while leaving a 1/8-inch space around the stoma

ANS: B Selecting a pouch that holds a large volume of output will decrease the frequency of emptying the pouch and may ease patient anxiety about pouch overflow. The barrier device should be changed every few days unless it is leaking or is no longer effective. Peristomal skin should be gently cleansed; vigorous rubbing can cause further irritation or skin breakdown. Approximately 1/16 of an inch is present between the barrier device and the stoma. Excess space allows fecal matter to have prolonged exposure to skin, resulting in skin breakdown.

8. Which patient is most at risk for increased peristalsis? a. A 5-year-old child who ignores the urge to defecate owing to embarrassment b. A 21-year-old patient with three final examinations on the same day c. A 40-year-old woman with major depressive disorder d. An 80-year-old man in an assisted-living environment

ANS: B Stress can stimulate digestion and increase peristalsis. Ignoring the urge to defecate, depression, and age-related changes of the elderly are causes of constipation.

13. Which nursing intervention is most effective in promoting normal defecation for a patient who has muscle weakness in the legs that prevents ambulation? a. Elevate the head of the bed 45 degrees 60 minutes after breakfast. b. Use a mobility device to place the patient on a bedside commode. c. Give the patient a pillow to brace against the abdomen while bearing down. d. Administer a soap suds enema every 2 hours.

ANS: B The best way to promote normal defecation is to assist the patient into a posture that is as normal as possible while defecating. Using a mobility device promotes nurse and patient safety. Elevating the head of the bed would be appropriate if the patient were to void with a bed pan. However, the patient's condition does not require use of a bed pan. Giving the patient a pillow may reduce discomfort, but this is not the best way to promote defecation. A soaps suds enema is indicated for a patient who needs assistance to stimulate peristalsis. It promotes non-natural defecation.

17. The nurse has attempted to administer a tap water enema for a patient with fecal impaction with no success. What is the next priority nursing action? a. Preparing the patient for a second tap water enema b. Donning gloves for digital removal of the stool c. Positioning the patient on the left side d. Inserting a rectal tube

ANS: B When enemas are not successful, digital removal of the stool may be necessary occasionally to break up pieces of the stool or to stimulate the anus to defecate. Tap water enemas should not be repeated because of risk of fluid imbalance. Positioning the patient on the left side does not promote defecation. A rectal tube is indicated for a patient with liquid stool incontinence but would not be applicable or effective for this patient.

After teaching a client who is recovering from a colon resection, the nurse assesses the clients understanding. Which statements by the client indicate a correct understanding of the teaching? (Select all that apply.) a. I must change the ostomy appliance daily and as needed. b. I will use warm water and a soft washcloth to clean around the stoma. c. I might start bicycling and swimming again once my incision has healed. d. Cutting the flange will help it fit snugly around the stoma to avoid skin breakdown. e. I will check the stoma regularly to make sure that it stays a deep red color. f. I must avoid dairy products to reduce gas and odor in the pouch.

ANS: B, C, D The ostomy appliance should be changed as needed when the adhesive begins to decrease, placing the appliance at risk of leaking. Changing the appliance daily can cause skin breakdown as the adhesive will still be secured to the clients skin. The client should avoid using soap to clean around the stoma because it might prevent effective adhesion of the ostomy appliance. The client should use warm water and a soft washcloth instead. The tissue of the stoma is very fragile, and scant bleeding may occur when the stoma is cleaned. The flange should be cut to fit snugly around the stoma to reduce contact between excretions and the clients skin. Exercise (other than some contact sports) is important for clients with an ostomy. The stoma should remain a soft pink color. A deep red or purple hue indicates ischemia and should be reported to the surgeon right away. Yogurt and buttermilk can help reduce gas in the pouch, so the client need not avoid dairy products.

28. The nurse administers a cathartic to a patient. The nurse determines that the cathartic has had a therapeutic effect when the patient a. Has a decreased level of anxiety. b. Experiences pain relief. c. Has a bowel movement. d. Passes flatulence.

ANS: C A cathartic is a laxative that stimulates a bowel movement. It would be effective if the patient experiences a bowel movement. The other options are not outcomes of administration of a cathartic

A nurse prepares a client for a colonoscopy scheduled for tomorrow. The client states, My doctor told me that the fecal occult blood test was negative for colon cancer. I dont think I need the colonoscopy and would like to cancel it. How should the nurse respond? a. Your doctor should not have given you that information prior to the colonoscopy. b. The colonoscopy is required due to the high percentage of false negatives with the blood test. c. A negative fecal occult blood test does not rule out the possibility of colon cancer. d. I will contact your doctor so that you can discuss your concerns about the procedure.

ANS: C A negative result from a fecal occult blood test does not completely rule out the possibility of colon cancer. To determine whether the client has colon cancer, a colonoscopy should be performed so the entire colon can be visualized and a tissue sample taken for biopsy. The client may want to speak with the provider, but the nurse should address the clients concerns prior to contacting the provider.

19. The nurse is preparing to perform a fecal occult blood test. The nurse plans to properly perform the examination by a. Applying liberal amounts of stool to the guaiac paper. b. Testing the quality control section before collecting the specimen section. c. Reporting any abnormal findings to the provider. d. Applying sterile disposable gloves.

ANS: C Abnormal findings such as a positive test should be reported to the provider. A fecal occult blood test is a clean procedure; sterile gloves are not needed. A thin specimen smear is all that is required. The quality control section should be developed after it is determined whether the sample is positive or negative.

16. The nurse would anticipate which diagnostic examination for a patient with black tarry stools? a. Ultrasound b. Barium enema c. Upper endoscopy d. Flexible sigmoidoscopy

ANS: C Black tarry stools are an indication of ulceration or bleeding in the upper portion of the GI tract; upper endoscopy would allow visualization of the bleeding. No other option would allow upper GI visualization.

A nurse teaches a client who is recovering from a colon resection. Which statement should the nurse include in this clients plan of care? a. You may experience nausea and vomiting for the first few weeks. b. Carbonated beverages can help decrease acid reflux from anastomosis sites. c. Take a stool softener to promote softer stools for ease of defecation. d. You may return to your normal workout schedule, including weight lifting.

ANS: C Clients recovering from a colon resection should take a stool softener as prescribed to keep stools a soft consistency for ease of passage. Nausea and vomiting are symptoms of intestinal obstruction and perforation and should be reported to the provider immediately. The client should be advised to avoid gas-producing foods and carbonated beverages, and avoid lifting heavy objects or straining on defecation.

25. A patient had an ileostomy surgically placed 2 days ago. Which diet would the nurse recommend to the patient to ease the transition of the new ostomy? a. Eggs over easy, whole wheat toast, and orange juice with pulp b. Chicken fried rice with stir fried vegetables and iced tea c. Turkey meatloaf with white rice and apple juice d. Fish sticks with macaroni and cheese and soda

ANS: C During the first week or so after ostomy placement, the patient should consume easy-to-digest low-fiber foods such as poultry, rice and noodles, and strained fruit juices. Fried foods can irritate digestion and can cause blockage. Foods high in fiber will be useful later in the recovery process but can cause blockage if the GI tract is not accustomed to digesting with an ileostomy.

A nurse teaches a client who is at risk for colon cancer. Which dietary recommendation should the nurse teach this client? a. Eat low-fiber and low-residual foods. b. White rice and bread are easier to digest. c. Add vegetables such as broccoli and cauliflower to your new diet. d. Foods high in animal fat help to protect the intestinal mucosa. .

ANS: C The client should be taught to modify his or her diet to decrease animal fat and refined carbohydrates. The client should also increase high-fiber foods and Brassica vegetables, including broccoli and cauliflower, which help to protect the intestinal mucosa from colon cancer

32. A nurse is caring for an older adult patient with fecal incontinence due to cathartic use. The nurse is most concerned about which complication that has the greatest risk for severe injury? a. Rectal skin breakdown b. Contamination of existing wounds c. Falls from attempts to reach the bathroom d. Cross-contamination into the upper GI tract

ANS: C The nurse is most concerned about the worst injury the patient could receive, which involves falling while attempting to get to the bathroom. To reduce injury, the nurse should clear the path and reinforce use of the call light. The question is asking for the greatest risk of injury, not the most frequent occurrence or the event most likely to occur.

A nurse assessing a client with colorectal cancer auscultates high-pitched bowel sounds and notes the presence of visible peristaltic waves. Which action should the nurse take? a. Ask if the client is experiencing pain in the right shoulder. b. Perform a rectal examination and assess for polyps. c. Contact the provider and recommend computed tomography. d. Administer a laxative to increase bowel movement activity.

ANS: C The presence of visible peristaltic waves, accompanied by high-pitched or tingling bowel sounds, is indicative of partial obstruction caused by the tumor. The nurse should contact the provider with these results and recommend a computed tomography scan for further diagnostic testing. This assessment finding is not associated with right shoulder pain; peritonitis and cholecystitis are associated with referred pain to the right shoulder. The registered nurse is not qualified to complete a rectal examination for polyps, and laxatives would not help this client.

A nurse assesses a male client with an abdominal hernia. Which abdominal hernias are correctly paired with their physiologic processes? (Select all that apply.) a. Indirect inguinal hernia An enlarged plug of fat eventually pulls the peritoneum and often the bladder into a sac b. Femoral hernia A peritoneum sac pushes downward and may descend into the scrotum c. Direct inguinal hernia A peritoneum sac passes through a weak point in the abdominal wall d. Ventral hernia Results from inadequate healing of an incision e. Incarcerated hernia Contents of the hernia sac cannot be reduced back into the abdominal cavity

ANS: C, D, E A direct inguinal hernia occurs when a peritoneum sac passes through a weak point in the abdominal wall. A ventral hernia results from inadequate healing of an incision. An incarcerated hernia cannot be reduced or placed back into the abdominal cavity. An indirect inguinal hernia is a sac formed from the peritoneum that contains a portion of the intestine and pushes downward at an angle into the inguinal canal. An indirect inguinal hernia often descends into the scrotum. A femoral hernia protrudes through the femoral ring and, as the clot enlarges, pulls the peritoneum and often the urinary bladder into the sac.

A nurse assesses a client with a mechanical bowel obstruction who reports intermittent abdominal pain. An hour later the client reports constant abdominal pain. Which action should the nurse take next? a. Administer intravenous opioid medications. b. Position the client with knees to chest. c. Insert a nasogastric tube for decompression. d. Assess the clients bowel sounds.

ANS: D A change in the nature and timing of abdominal pain in a client with a bowel obstruction can signal peritonitis or perforation. The nurse should immediately check for rebound tenderness and the absence of bowel sounds. The nurse should not medicate the client until the provider has been notified of the change in his or her condition. The nurse may help the client to the knee-chest position for comfort, but this is not the priority action. The nurse need not insert a nasogastric tube for decompression.

11. Fecal impactions occur in which portion of the colon? a. Ascending b. Descending c. Transverse d. Rectum

ANS: D A fecal impaction is a collection of hardened feces wedged in the rectum that cannot be expelled. It results from unrelieved constipation. Feces at this point in the colon contain the least amount of moisture. Feces found in the ascending, transverse, and descending colon still consist mostly of liquid and do not form a hardened mass.

38. A guaiac test has been ordered. The nurse knows that this is a test for a. Bright red blood. b. Dark black blood. c. Blood that contains mucus. d. Blood that cannot be seen.

ANS: D Fecal occult blood tests are used to test for blood that may be present in stool that cannot be seen by the naked eye. This is usually indicative of a GI bleed. All other options are incorrect.

33. The nurse is caring for a patient with Clostridium difficile. Which of the following nursing actions will have the greatest impact in preventing the spread of bacteria? a. Monthly in-services about contact precautions b. Placing all contaminated items in biohazard bags c. Mandatory cultures on all patients d. Proper hand hygiene techniques

ANS: D Proper hand hygiene is the best way to prevent the spread of bacteria. Monthly in-services place emphasis on education, not on action. Biohazard bags are appropriate but cannot be used on every item that C. difficile comes in contact with, such as a human. Mandatory cultures are expensive and unnecessary and would not prevent the spread of bacteria.

A nurse cares for a client who has a family history of colon cancer. The client states, My father and my brother had colon cancer. What is the chance that I will get cancer? How should the nurse respond? a. If you eat a low-fat and low-fiber diet, your chances decrease significantly. b. You are safe. This is an autosomal dominant disorder that skips generations. c. Preemptive surgery and chemotherapy will remove cancer cells and prevent cancer. d. You should have a colonoscopy more frequently to identify abnormal polyps early.

ANS: D The nurse should encourage the client to have frequent colonoscopies to identify abnormal polyps and cancerous cells early. The abnormal gene associated with colon cancer is an autosomal dominant gene mutation that does not skip a generation and places the client at high risk for cancer. Changing the clients diet, preemptive chemotherapy, and removal of polyps will decrease the clients risk but will not prevent cancer. However, a client at risk for colon cancer should eat a low-fat and high-fiber diet.

A client is scheduled for discharge after surgery for inflammatory bowel disease. The client's spouse will be assisting home health services with the client's care. What is most important for the home health nurse to assess in the client and the spouse with regard to the client's home care? A. Ability of the client and spouse to perform incision care and dressing changes B. Effective coping mechanisms for the client and spouse after the surgical experience C. Knowledge about the client's requested pain medications D. Understanding of the importance of keeping scheduled follow-up appointments

Ability of the client and spouse to perform incision care and dressing changes Assessing the client's and the spouse's ability to carry out incision care and dressing changes is essential for avoiding further development of the infectious process, as well as infection of the surgical incision itself. Assessing coping mechanisms and knowledge of the client's pain medication are important, but are not the priority. Understanding the importance of scheduled follow-up appointments is important, but is not the priority.

A client with gastric cancer is scheduled to undergo surgery to remove the tumor once 5 pounds of body weight has been regained. The client is not drinking the vanilla-flavored enteral supplements that have been prescribed. Which is the highest priority nursing intervention for this client? A. Explain to the client the importance of drinking the enteral supplements prescribed. B. Ask the client's family to try to persuade the client to drink the supplements. C. Inform the client that a nasogastric tube may be necessary if he or she fails to comply. D. Ask the client if a change in flavor would make the supplement more palatable.

Ask the client if a change in flavor would make the supplement more palatable. Asking the client if a change in flavor would help shows that the nurse is attempting to determine why the client is not drinking the supplements. Many clients don't like certain supplement flavors. The nurse should not assume that the client does not understand the importance of drinking the supplements or that the client requires persuasion to drink the supplements. The problem may be entirely different. Telling the client that a nasogastric tube may be necessary could be construed as threatening the client.

The nurse prepares a teaching session regarding lifestyle changes needed to decrease the discomfort associated with a client's hiatal hernia. Which change does the nurse recommend to this client? A. Eat only two or three meals daily. B. Sleep flat in a left side-lying position. C. Drink tea instead of coffee. D. Avoid working while bent over the computer.

Avoid working while bent over the computer. The client should avoid working while bent over because this position presses on the diaphragm, causing discomfort. The client with a hiatal hernia should eat four to six meals a day. The head of the client's bed should be elevated approximately 6 inches. Both tea and coffee should be eliminated from this client's diet because of the caffeine content.

The nurse is caring for a client with esophageal cancer who has received photodynamic therapy using porfimer sodium (Photofrin). What instructions does the nurse include in teaching the client about porfimer sodium? (Select all that apply.) A. Avoid sunlight for 2 weeks. B. Cover all exposed body areas. C. Follow a clear liquid diet for 3 to 5 days after the procedure. D. Monitor for hypertension. E. Tissue particles may be found in the sputum.

B. Cover all exposed body areas. C. Follow a clear liquid diet for 3 to 5 days after the procedure. E. Tissue particles may be found in the sputum. Porfimer sodium causes photosensitivity, and sunglasses and protective clothing covering all exposed body areas are essential. A clear liquid diet should be followed for 3 to 5 days after the procedure and then should be advanced to full liquids as tolerated. The client should be warned that tissue particles may be released from the tumor site and may be present in the sputum. Sunlight should be avoided for 1 to 3 months. Side effects are rare and may include nausea, fever, and constipation. Hypertension is not a side effect of porfimer sodium.

The nurse is assessing a client with gastroesophageal reflux disease (GERD). Which findings does the nurse expect to observe? (Select all that apply.) A. Blood-tinged sputum B. Dyspepsia C. Excessive salivation D. Flatulence E. Regurgitation

B. Dyspepsia D. Flatulence E. Regurgitation Dyspepsia, also known as heartburn, is one of the main symptoms of GERD. Flatulence is common after eating, as well as regurgitation (backward flow into the throat) of food and fluids. Blood-tinged sputum and excessive salivation are not symptoms of GERD.

A client has undergone conventional esophageal surgery. The client's diet has been advanced to semi-solid, and feedings are well tolerated. The client reports experiencing diarrhea about 1 hour after each meal. What is the priority nursing intervention to help prevent further diarrhea? A. Ensure that the client takes adequate amounts of fluids with meals. B. Advance the diet to solid food and encourage eating as much as possible at meals. C. Give the client a dose of magnesium hydroxide (Milk of Magnesia) after each meal. D. Encourage the client to take fluids between meals rather than with meals.

Encourage the client to take fluids between meals rather than with meals. Diarrhea is believed to be the result of vagotomy syndrome and can be managed by taking fluids between meals rather than with meals. For this client, fluids with meals can lead to the development of diarrhea immediately after eating. The client may not be physically ready to advance to a solid diet. The client should eat six to eight small meals daily. Magnesium hydroxide is a magnesium-based antacid that can cause diarrhea.

A home health client has had severe diarrhea for the past 24 hours. Which nursing action does the RN delegate to the home health aide (unlicensed assistive personnel [UAP]) who assists the client with self-care? A. Instructing the client about the use of electrolyte-containing oral rehydration products B. Administering loperamide (Imodium) 4 mg from the client's medicine cabinet C. Checking and reporting the client's heart rate and blood pressure in lying, sitting, and standing positions D. Teaching the client how to clean the perineal area after each loose stool

Checking and reporting the client's heart rate and blood pressure in lying, sitting, and standing positions Obtaining the client's blood pressure and heart rate is included in the education of home health aides and other UAP. Client teaching and medication administration are complex skills that should be performed by licensed nurses who have the education and scope of practice needed to safely implement these actions.

The nurse and the dietitian are planning sample diet menus for a client who is experiencing dumping syndrome. Which sample meal is best for this client? A. Chicken salad on whole wheat bread B. Liver and onions C. Chicken and rice D. Cobb salad with buttermilk ranch dressing

Chicken and rice Chicken and rice is the only selection suitable for the client who is experiencing dumping syndrome because it contains high protein without the addition of milk or wheat products. The client with dumping syndrome should not be allowed to have mayonnaise, onions, or buttermilk ranch dressing; the dressing is made from milk products. The client can have whole wheat bread only in very limited amounts.

A client has vague symptoms that indicate an acute inflammatory bowel disorder. Which symptom is most indicative of Crohn's disease (CD)? A. Abdominal pain relieved by bending the knees B. Chronic diarrhea, abdominal pain, and fever C. Epigastric cramping D. Hypotension with vomiting

Chronic diarrhea, abdominal pain, and fever Chronic diarrhea, abdominal pain, and fever are symptoms more indicative of CD than of other acute inflammatory bowel disorders. Abdominal pain that is relieved by bending the knees is indicative of peritonitis or pancreatitis. Epigastric cramping is a symptom more indicative of appendicitis. Hypotension with vomiting is not characteristic of CD.

Which is a correct statement differentiating Crohn's disease (CD) from ulcerative colitis (UC)? A. Clients with CD experience about 20 loose, bloody stools daily. B. Clients with UC may experience hemorrhage. C. The peak incidence of UC is between 15 and 40 years of age. D. Very few complications are associated with CD.

Clients with UC may experience hemorrhage. Hemorrhage is commonly experienced by clients with UC. Five to six stools daily is common with CD. The peak incidences of UC are between 15 to 25 and 55 to 65 years of age. Fistulas commonly occur as a complication of CD.

A client with ulcerative colitis is prescribed sulfasalazine (Azulfidine) and corticosteroid therapy. As the disease improves, what change does the nurse expect in the client's medication regimen? A. Corticosteroid therapy will be stopped. B. Sulfasalazine (Azulfidine) will be stopped. C. Corticosteroid therapy will be tapered. D. Sulfasalazine (Azulfidine) will be tapered.

Corticosteroid therapy will be tapered. Once clinical improvement has been established, corticosteroids are tapered over a 2- to 3-month period. Stopping corticosteroid therapy abruptly is unsafe—steroids must be gradually decreased in clients. Usually the amount that they have been taking dictates how quickly or slowly they can be stopped. Sulfasalazine therapy will be taken on a long-term basis. It may be increased or decreased, depending on the client's symptoms, but will likely never be stopped. These decisions are made over a long period of therapy.

The nurse is working with the dietitian to plan a menu for a client who has persistent difficulty swallowing. What is a suitable breakfast selection for this client? A. Scrambled eggs and toast B. Oatmeal and orange juice C. Puréed fruit and English muffin D. Cream of wheat and applesauce

Cream of wheat and applesauce Both cream of wheat and applesauce are foods of semi-solid consistency and are appropriate for this client. The client who is having difficulty swallowing should be given semi-solid foods and thickened liquids. Toast would not be appropriate, and orange juice would have to be thickened before it is given to this client. An English muffin would be inappropriate for this client because it is not a semi-solid food.

A client with a recent surgically created ileostomy refuses to look at the stoma and asks the nurse to perform all required stoma care. What does the nurse do next? A. Asks the client whether family members could be trained in stoma care B. Has another client with a stoma who performs self-care talk with the client C. Requests that the health care provider request antidepressants and a psychiatric consult D. Suggests that the health care provider request a home health consultation so stoma care can be performed by a home health nurse

Has another client with a stoma who performs self-care talk with the client Talking with another client who successfully cares for his or her stoma may give the client the confidence to begin his or her self-care. If at all possible, the client should perform stoma care so that he or she can be as independent as possible. Although the client may need medication for depression, the priority is to encourage the client to look at, touch, and begin caring for the stoma. A home health nurse can be a support, but cannot provide all of the care that the client will need.

A client with a history of osteoarthritis has a 10-inch incision following a colon resection. The incision has become infected, and the wound requires extensive irrigation and packing. What aspect of the client's care does the nurse make certain to discuss with the health care provider before the client's discharge? A. Having a home health consultation for wound care B. Requesting an antianxiety medication C. Requesting pain medication for the client's osteoarthritis D. Placing the client in a skilled nursing facility for rehabilitation

Having a home health consultation for wound care Home health services are most appropriate for this client because wound care will be extensive and the client's mobility may be limited. No indication suggests that the client is experiencing anxiety regarding postoperative care. Pain medication may be needed for the client's osteoarthritis, but this is not the highest priority. A skilled nursing facility is not necessary if the client can remain in his or her home with sufficient support services.

The nurse is reviewing orders for a client with possible esophageal trauma after a car crash. Which request does the nurse implement first? A. Give total parenteral nutrition (TPN) through a central venous catheter. B. Administer cefazolin (Kefzol) 1 g intravenously. C. Obtain a computed tomography (CT) scan of the chest and abdomen. D. Keep the client nothing by mouth (NPO) for possible surgery.

Keep the client nothing by mouth (NPO) for possible surgery. Clients with possible esophageal tears should be NPO until diagnostic testing is completed, because leakage of anything taken orally into the sterile mediastinum could occur. In addition, esophageal rest is maintained for about 10 days after esophageal trauma to allow time for mucosal healing. TPN is prescribed to provide calories and protein for wound healing; although this is important, it is not a priority for the nurse to implement first. Antibiotics may be requested to prevent possible infection, but this is not the priority. A CT of the chest and abdomen will be needed, but is not the nurse's initial action.

A client with oral cancer is depressed over the diagnosis and tells the nurse of plans to have a radical neck dissection. What is the nurse's best reaction? A. Listen to the client and then explain that it is normal to feel depressed about the diagnosis. B. Explain the grieving process and listen to what the client has to say. C. Suggest that the client talk with friends and family and seek their support. D. Listen to the client's concerns and feelings, and then suggest that the client join a community group of cancer survivors.

Listen to the client's concerns and feelings, and then suggest that the client join a community group of cancer survivors. The nurse should listen to the client and suggest a community support group of those with similar diagnoses who can offer support to the client. Telling the client that his or her feelings are normal or explaining the grieving process to the client are not helpful or therapeutic; the client needs more guidance. The nurse should not assume that the client's family and friends are an appropriate support group, because this may not be the case.

The nurse is caring for a client diagnosed with esophageal cancer who is experiencing diarrhea after conventional esophageal surgery. The nurse anticipates that the health care provider will request which medication to manage diarrhea? A. Loperamide (Imodium) B. Mesalamine (Pentasa) C. Minocycline (Minocin) D. Pantoprazole (Protonix)

Loperamide (Imodium) Diarrhea is thought to be the result of vagotomy syndrome, which develops as a result of interruption of vagal fibers to the abdominal viscera during surgery. It can occur 20 minutes to 2 hours after eating and can be symptomatically managed with loperamide. Mesalamine is used to treat clients with mild to moderate ulcerative colitis. Minocycline is an antibiotic used for treatment of infection. Pantoprazole is used to treat gastroesophageal reflux disease.

A client admitted with severe gastroenteritis has been started on an IV, but the client continues having excessive diarrhea. Which medication does the nurse ask the health care provider about prescribing? A. Balsalazide (Colazal) B. Loperamide (Imodium) C. Mesalamine (Asacol) D. Milk of Magnesia (MOM)

Loperamide (Imodium) If the health care provider determines that antiperistaltic agents are necessary, an initial dose of loperamide (Imodium) 4 mg can be administered orally, followed by 2 mg after each loose stool, up to 16 mg daily. Balsalazide is not the best choice for control of diarrhea in this scenario. Mesalamine is used for clients with ulcerative colitis for long-term therapy. MOM is a laxative.

A client has recently developed acute sialadenitis. Which intervention does the nurse include in this client's care? A. Applying cold compresses B. Avoiding the use of fruit or citrus-flavored candy C. Massaging the salivary gland D. Keeping the head of the bed at 10 degrees when the client is lying down

Massaging the salivary gland The salivary gland is massaged to stimulate the flow of saliva. This is done by milking the edematous gland with the fingertips toward the ductal opening. To promote the flow of saliva, warm compresses are applied to the affected salivary gland. Sialagogues such as lemon slices and fruit- or citrus-flavored candy are used to stimulate the flow of saliva. Elevation of the head of the bed promotes gravity drainage of the edematous gland.

The nurse reviews a medication history for a client newly diagnosed with peptic ulcer disease (PUD) who has a history of using ibuprofen (Advil) frequently for chronic knee pain. The nurse anticipates that the health care provider will request which medication for this client? A. Bismuth subsalicylate (Pepto-Bismol) B. Magnesium hydroxide (Maalox) C. Metronidazole (Flagyl) D. Misoprostol (Cytotec)

Misoprostol (Cytotec) Misoprostol is a prostaglandin analogue that protects against nonsteroidal anti-inflammatory drug (NSAID)-induced ulcers. Bismuth subsalicylate is an antidiarrheal drug that contains salicylates, which can cause bleeding and should be avoided in clients who have PUD. Magnesium hydroxide is an antacid that may be used to neutralize stomach secretions, but is not used specifically to help prevent NSAID-induced ulcers. Metronidazole is an antimicrobial agent used to treat Helicobacter pylori infection.

The nurse is caring for a postoperative client who had a radical neck dissection, and the client is describing throbbing pain in the head. The nurse anticipates that the health care provider will request which medication for this client? A. Diphenhydramine (Benadryl) B. Midazolam (Versed) intravenously C. Morphine sulfate intravenously D. Oxycodone plus acetaminophen (Percocet, Tylox)

Morphine sulfate intravenously Clients undergoing surgery for oral cancer describe their pain as throbbing or pounding. Intravenous morphine sulfate is indicated for severe pain and is given initially. Diphenhydramine is an anti-inflammatory agent and is not indicated for treatment of pain. Midazolam is used for conscious sedation and is not indicated for pain. Oxycodone/acetaminophen is given for systematic relief of moderate pain.

The nurse is reinforcing the instructions on swallowing provided by the speech-language pathologist to a client diagnosed with esophageal cancer. Which instruction to the client is the highest priority? A. Place food at the back of the mouth as you eat. B. Do not be overly concerned with tongue or lip movements. C. Before swallowing, tilt the head back to straighten the esophagus. D. Do not attempt to reach food particles that are on the lips or around the mouth.

Place food at the back of the mouth as you eat. Placing food at the back of the mouth when eating will help the client avoid aspirating. Both tongue movements and sealing of the lips should be monitored in this client. The client's head should be tilted forward in the chin-tuck position. The client should be able to reach food particles on her or his lips and around the mouth with the tongue.

A client has had a radical neck dissection with a permanent tracheostomy for treatment of oral cancer. In what order should the following orders for postoperative nutritional care be implemented? A. Monitor weight, teach swallowing exercises, assess aspiration risk, provide nasogastric nutrition. B. Teach swallowing exercises, assess aspiration risk, monitor weight, provide nasogastric nutrition. C. Assess aspiration risk, teach swallowing exercises, provide nasogastric nutrition, monitor weight. D. Provide nasogastric nutrition, assess aspiration risk, monitor weight, teach swallowing exercises.

Provide nasogastric nutrition, assess aspiration risk, monitor weight, teach swallowing exercises. Clients who have undergone radical neck dissection with a permanent tracheostomy are nothing by mouth (NPO) for several days. Nasogastric and total parental nutrition will be needed until oral nutrition can begin. Next, the nurse should assess the client's ability to swallow or should note signs of aspiration when oral intake is started. Third, weights and hydration should be monitored daily, because the client may need additional nutritional supplements. Last, the client should perform swallowing exercises and work with a speech-language pathologist to learn swallowing techniques.

A client is scheduled to be discharged after a gastrectomy. The client's spouse expresses concern that the client will be unable to change the surgical dressing adequately. What is the nurse's highest priority intervention? A. Providing both oral and written instructions on changing the dressing and on symptoms of infection that must be reported to the provider B. Asking the provider for a referral for home health services to assist with dressing changes C. Asking the spouse whether other family members could be taught how to change the dressing D. Trying to determine specific concerns that the spouse has regarding dressing changes

Providing both oral and written instructions on changing the dressing and on symptoms of infection that must be reported to the provider Providing the client and spouse with both oral and written instructions on symptoms to report to the provider, as well as on how to perform the dressing change, will reinforce important points and boost the spouse's confidence. Obtaining a referral and recruiting other family members prevent the client and spouse from taking responsibility for the client's care. The spouse's concerns have already been clearly expressed.

Which nursing action is best for the charge nurse to delegate to an experienced LPN/LVN? A. Retape the nasogastric tube for a client who has had a subtotal gastrectomy and vagotomy. B. Reinforce the teaching about avoiding alcohol and caffeine for a client with chronic gastritis. C. Document instructions for a client with chronic gastritis about how to use "triple therapy." D. Assess the gag reflex for a client who has arrived from the postanesthesia care unit after a laparoscopic gastrectomy.

Reinforce the teaching about avoiding alcohol and caffeine for a client with chronic gastritis. Reinforcement of teaching done by the RN is within the scope of practice for an LPN/LVN. Retaping the nasogastric tube for a client who has had a subtotal gastrectomy and vagotomy is a complex task that should be done by the RN. Assessment and documenting instructions about how to use triple therapy are nursing functions that should be done by the RN.

A client is being discharged after a minimally invasive esophagectomy. Which teaching point does the nurse consider to be of the highest priority during the predischarge teaching session? A. Instruct the client to eat three meals daily. B. Emphasize the importance of lying down after meals. C. Encourage the client to ask his or her health care provider for antidepressant medication. D. Report the presence of fever and a swollen, painful neck incision.

Report the presence of fever and a swollen, painful neck incision. Wound management and prevention of infection are major concerns because the client who has had an esophagectomy typically has multiple drains and incisions. The client should eat six to eight small meals daily, and should sit up after meals to encourage satisfactory swallowing. The client's coping skills should be assessed, as well as his or her level of anxiety and/or depression, before antidepressant medication is prescribed.

The nurse is providing instructions to a client who has a history of stomatitis. Which instructions does the nurse include in the client's teaching plan? A. Encourage the client to eat acidic foods to decrease bacteria. B. Mouth care should be performed twice daily. C. Rinse the mouth with warm saline or sodium bicarbonate. D. Use a medium-bristled toothbrush for oral care.

Rinse the mouth with warm saline or sodium bicarbonate. Rinsing the mouth with warm saline or sodium bicarbonate or a combination of the two decreases inflammation and pain. Acidic foods increase inflammation and should be avoided. Mouth care should be done after each meal and as often as needed. If stomatitis is not controlled, mouth care may have to be done every 2 hours or more frequently. A soft toothbrush should be used for oral care.

A client returns to the unit after having an exploratory abdominal laparotomy. How does the nurse position this client after the client is situated in bed? A. High Fowler's B. Lateral Sims' (side-lying) C. Semi-Fowler's D. Supine

Semi-Fowler's The client is maintained in semi-Fowler's position to facilitate the drainage of peritoneal contents into the lower region of the abdominal cavity after an abdominal laparotomy. This position also helps increase lung expansion. High-Fowler's position would be too high for the client postoperatively; it would place strain on the abdominal incision(s), and, if the client was still drowsy from anesthesia, this position would not enhance the client's ability to rest. Sims' position does not promote drainage to the lower abdomen. The supine position does not facilitate drainage to the abdomen or increased lung expansion; the client would be more likely to develop complications (wound drainage stasis and atelectasis) in this position.

An 80-year-old client with a 2-day history of myalgia, nausea, vomiting, and diarrhea is admitted to the medical-surgical unit with a diagnosis of gastroenteritis. Which health care provider request does the nurse implement first? A. Administer acetaminophen (Tylenol) 650 mg rectally. B. Draw blood for a complete blood count and serum electrolytes. C. Obtain a stool specimen for culture and sensitivity. D. Start an IV solution of 5% dextrose in 0.45 normal saline at 125 mL/hr.

Start an IV solution of 5% dextrose in 0.45 normal saline at 125 mL/hr. Fluid therapy is the focus of treatment for clients with gastroenteritis. Older clients are at increased risk for the complications of dehydration such as hypovolemia and acute kidney failure. Acetaminophen 650 mg should be rapidly administered rectally, and blood draws and stool specimen collection should be implemented rapidly, but prevention and treatment of dehydration are the priorities for this client.

A client is experiencing bleeding related to peptic ulcer disease (PUD). Which nursing intervention is the highest priority? A. Starting a large-bore IV B. Administering IV pain medication C. Preparing equipment for intubation D. Monitoring the client's anxiety level

Starting a large-bore IV A large-bore IV should be placed as requested, so that blood products can be administered. IV pain medication is not a recommended treatment for gastrointestinal bleeding. Intubation is not a recommended treatment for bleeding related to PUD. The mental status of the client should be monitored, but it is not necessary to monitor the anxiety level of the client.

An obese client is discharged 10 days after being hospitalized for peritonitis, which resulted in an exploratory laparotomy. Which assessment finding by the client's home health nurse requires immediate action? A. Pain when coughing B. States, "I am too tired to walk very much" C. States, "I feel like the incision is splitting open" D. Temperature of 100.8° F (38.2° C).

States, "I feel like the incision is splitting open" The client feeling like the incision is splitting open is at risk for poor wound healing and possible wound dehiscence; the nurse should immediately assess the wound and notify the health care provider. Reports of pain when coughing, being too tired to ambulate, and a temperature of 100.8° F (38.2° C) all require further assessment or intervention, but are not as great a concern as the possibility of wound dehiscence for this client.

A client who has undergone surgery and radiation therapy to treat oral cancer reports persistent dry mouth. What will the nurse teach this client about managing this symptom? A. Use saliva substitutes, especially when eating dry foods. B. This condition is common but is temporary. C. Use lozenges and hard candies to prevent dry mouth. D. This indicates a complication of therapy.

Use saliva substitutes, especially when eating dry foods. Xerostomia is a common effect of oral irradiation and may be permanent. Clients should be advised to use saliva substitutes. The condition is common, but often permanent. Lozenges and hard candies are not as effective as saliva substitutes. Dry mouth is a side effect of therapy, not a symptom of complications.

The nurse is caring for a client with a hiatal hernia who had an open fundoplication yesterday. Which task does the nurse delegate to unlicensed assistive personnel (UAP)? A. Using a pillow to support the incision when the client coughs B. Adjusting the position of the nasogastric (NG) tube C. Assessing the level of postoperative pain using a 0-to-10 scale D. Giving the client sips of water once bowel sounds are heard

Using a pillow to support the incision when the client coughs Assisting a client to cough is a task within the education and skill level of UAP. NG tube maintenance, pain assessment, and assessment of bowel sounds require more knowledge of the potential complications associated with this surgical procedure, and are actions best performed by licensed nursing staff.

A client has an anal fissure. Which intervention most effectively promotes perineal comfort for the client? A. Administering a Fleet's enema when needed B. Applying heat to acute inflammation for pain relief C. Avoiding the use of bulk-forming agents D. Using hydrocortisone cream to relieve pain

Using hydrocortisone cream to relieve pain Witch hazel wipes may be effective in relieving the pain associated with anal fissures. Enemas should be avoided when an anal fissure is present. Cold packs should be applied to acute inflammation to diminish discomfort. Bulk-forming agents should be used to decrease pain associated with defecation.

A client who had surgery for inflammatory bowel disease is being discharged. The case manager will arrange for home health care follow-up. The client tells the nurse that family members will also be helping with care. What information is critically important for the nurse to provide to these collaborating members? A. A list of medical supply facilities where wound care supplies may be purchased B. Proper handwashing techniques to avoid cross-contamination of the client's wound C. The amount of pain medication that the client is allowed to take in each dose D. Written and oral instructions regarding symptoms to report to the health care provider

Written and oral instructions regarding symptoms to report to the health care provider It is most important to provide the client and case manager with both written and oral instructions on reportable symptoms to avoid the development of complications. Although instruction on proper handwashing and the client's medication regimen are important, they are not the highest priority. It will be the home health nurse's responsibility to bring supplies to the client's home.


Ensembles d'études connexes

BIO201 Exam 2: Practice Questions

View Set

Developmental Psychology Final - NYU UG Psychology (Aalai, Fall 2022)

View Set

power point questions foundations

View Set

A The nature of management, B Strategic management & planning, C Organizational structure and design, D Culture, E Networking, F Motivation, G Individual decision making, H Group DM, I Leadership, J Organizational Change, K Issue/selling, Team group...

View Set